You are on page 1of 81

CRIMINAL PROCESS CAN

JURISDICTION.............................................................................................................................................2
1. legislative division of powers.....................................................................................................................2
2. classiciation of offences..............................................................................................................................3
indictable................................................................................................................................................................................... 3
summary.................................................................................................................................................................................... 5
hybrid......................................................................................................................................................................................... 6
process – classification of offences................................................................................................................................... 6
interpretation act..................................................................................................................................................................... 6
Choice of forum – short form............................................................................................................................................. 6
counsel appoints...................................................................................................................................................................... 7
jurisdiction of courts......................................................................................................................................7
time limitations........................................................................................................................................................................ 8
territorial limitations.............................................................................................................................................................. 8
jurisdiction under the charter............................................................................................................................................ 10
search and seizure.........................................................................................................................................12
1. POWERS TO SEARCH AND SEIZE....................................................................................................12
charter s.8 + Historical background................................................................................................................................ 12
(A) CRIMINAL CODE................................................................................................................................12
(i) without a warrant....................................................................................................................................12
(ii) with a search warrant.............................................................................................................................13
(B) COMMON LAW...................................................................................................................................16
(i) search incident to arrest..........................................................................................................................17
(ii) on consent...............................................................................................................................................22
(iii) Incident to duties of police officers – ancillary powers doctrine......................................................23
(c) other federal statutes.............................................................................................................................26
Controlled Drugs and Substances act ss.11, 12........................................................................................................... 26
(d) provincial statutes...................................................................................................................................26
(e) Constitutional minimum standards.......................................................................................................26
(i) Warrant required............................................................................................................................................................. 26
(ii) Warrantless searches.................................................................................................................................................... 27
(iIi) unlawful searches violate s.8................................................................................................................................... 29
(iV) Reasonable expectation of privacy........................................................................................................................ 30
1. POWERS OF ANY INDIVIDUAL.........................................................................................................34
(A) information before a justice: summons or arrest................................................................................34
(b) arrest without warrant...........................................................................................................................35
Criminal code s.494(1)....................................................................................................................................................... 36
2. SPECIAL POWER OF ARREST OF OWNERS, POSSESSORS AND THOSE THEY
AUTHORIZE................................................................................................................................................36
Criminal code 494(2).......................................................................................................................................................... 36
3. POWERS OF PEACE OFFICERS........................................................................................................36
(a) Breach of the peace.................................................................................................................................36
Criminal code s.31............................................................................................................................................................... 36
(b) arrest without warrant...........................................................................................................................36
Criminal code s.495............................................................................................................................................................ 36
(c) constitutional minimum standards........................................................................................................38
charter ss. 9, 10(a)................................................................................................................................................................ 38
(d) stop powers and racial profiing.............................................................................................................38
(i) vehicle stops..............................................................................................................................................38
highway traffic act s.216.................................................................................................................................................... 39
(ii) Investigative detention...........................................................................................................................39
(iIi) roadblock stops......................................................................................................................................41
(e) reasons for arrest....................................................................................................................................42
Criminal code s.29............................................................................................................................................................... 42
(f) entry into premises..................................................................................................................................42
Criminal code s.529.3......................................................................................................................................................... 42
(g) meaning of arrest....................................................................................................................................44
interrogation.................................................................................................................................................44
introduction............................................................................................................................................................................ 44
1. RIGHT TO COUNSEL CHARTER S.10(B).........................................................................................45
(a) triggering mechanism............................................................................................................................45
(b) informational duties...............................................................................................................................47
(c) Implementational duties.........................................................................................................................49
(d) waiver and duty to be reasonably diligent in exercise of right...........................................................53
2. right to silence...........................................................................................................................................56
(a) voluntary confessions rule prior to r v oickle.......................................................................................56
introduction............................................................................................................................................................................ 56
Salhany and carter “future of the law of evidence”................................................................................................... 57
(b) principles of fundamental justice (ch. s.7)............................................................................................57
(c) non-compellability (s.11(c)) and privielege against self incrimination (s.13)....................................59
(d) Principle against self incrimination (s.7)..............................................................................................60
(E) silence at trial..........................................................................................................................................60
(f) re-stated confessions rule r v oickle.......................................................................................................61
New common law in connection with Mr. Big Operations.......................................................................65

JURISDICTION

1. LEGISLATIVE DIVISION OF POWERS


CONSTITUIONAL DIVISION:

FEDERAL GOVT:
 Authority to make criminal law
 Authority to make criminal procedure law
o Mostly in criminal code
 Narcotics control Act CDSA
 Youth criminal justice Act

PROVINCIAL GOVTS:
 Provincial offences
o Provincial offences  authority to govern criminal procedure via the Provincial Offences
Procedure Act
 Administration of federal criminal justice in the provinces:
o Provincial attorney general  provincial jurisdiction for administration of justice

WHO PROSECUTES WHAT?


 Provincial crown prosecutes provincial code offences
 Federal crown also prosecutes Aeronautics, Narcotics, Terrorist offences
 Provincial crown prosecutes provincial offences
 Provincial crown also prosecutes young offenders
o Young Offenders Act = Federal legislation
o Federal gov’t has the authority to prosecute it but has delegated it to the crown

1) Criminal code (federal) – Provincial prosecutor


2) Drugs  federal prosecutor
3) Youth  provincial prosecutor
4) Drugs  federal prosecutor
5) Other federal legislation  federal prosecutor
a. EG – Aeronautics act

2. CLASSICIATION OF OFFENCES
4 types of offences
 Summary
 Indictable
 Hybrid
 Regulatory

INDICTABLE
 Chart 3:
o Date set for prelim  have to chose to do one
BASICS:
 More serious
 If not stated, the sentence is max 5 years imprisonment as per s.743 CC
 Can appear in provincial court
 ANY SUPERIOR COURT CAN HEAR:
o S.468: Every superior court of criminal justice has jurisdiction to try any indictable
offence

Look to EXCLUSIVE  ASBOLUTE  process of elimination  ELECTABLE INDICTABLE

ACCUSED PERSONALLY PRESENT IN COURT ROOM


 S.650Where an accused in charged with an indictable offence, he must be personally present at
all stages of the proceedings (accused other than an organization)
o Exceptions:
 S.650(1.1) CCTV
 S.650(2)- accused removed due to misconduct, if court considers it proper, unfit to
stand trial
 S.650.01 –can appoint/ designation of counsel of record filed with the court

3 TYPES:

1) Exclusive jurisdiction of superior courts (s.469)


o PROCESS: CHOICE OF FORUM
 First appearance in PC
 No election here
 Only question is whether you want a preliminary inquiry  almost always yes
 Preliminary inquiry in PC
 If Requested 536(4)
 Here can elect whether want judge and jury or just judge
 Trial in PC (s.536(3))
 Trial in QB (s.536(4))
 Judge and jury or Just judge
 Guilty plea or trial

 S.469: Exclusive jurisdiction offences:


 (i) section 47 (treason),
 (ii) section 49 (alarming Her Majesty),
 (iii) section 51 (intimidating Parliament or a legislature),
 (iv) section 53 (inciting to mutiny),
 (v) section 61 (seditious offences),
 (vi) section 74 (piracy),
 (vii) section 75 (piratical acts), or
 (viii) section 235 (murder)

2) Absolute jurisdiction offences


 PROCESS: CHOICE OF FORUM:
 First appearance PC
 No election
 Can’t decide between judge/jury
 Everything will be in PC
 No preliminary inquiry

 S.553 – Have to be heard by PC if charged (a) with:


 (i) theft, other than theft of cattle,
 (ii) obtaining money or property by false pretences
 (iii) unlawfully having in his possession any property or thing or any proceeds of any
property or thing knowing that all or a part of the property or thing or of the proceeds
was obtained by or derived directly or indirectly from the commission in Canada of an
offence punishable by indictment or an act or omission anywhere that, if it had occurred
in Canada, would have constituted an offence punishable by indictment,
 (iv) having, by deceit, falsehood or other fraudulent means, defrauded the public or any
person, whether ascertained or not, of any property, money or valuable security, or
 (v) mischief under subsection 430(4),where the subject-matter of the offence is not a
testamentary instrument and the alleged value of the subject-matter of the offence does
not exceed five thousand dollars;
 (b) with counselling or with a conspiracy or attempt to commit or with being an
accessory after the fact to the commission of
 (i) any offence referred to in paragraph (a) in respect of the subject-matter and value
thereof referred to in that paragraph, or
 (ii) any offence referred to in paragraph (c); or
 (c) with an offence under
 (i) section 201 (keeping gaming or betting house),
 (ii) section 202 (betting, pool-selling, book-making, etc.),
 (iii) section 203 (placing bets),
 (iv) section 206 (lotteries and games of chance),
 (v) section 209 (cheating at play),
 (vi) section 210 (keeping common bawdy-house),
 (vii) [Repealed, 2000, c. 25, s. 4]
 (viii) section 393 (fraud in relation to fares),
 (viii.01) section 490.031 (failure to comply with order or obligation),
 (viii.02) section 490.0311 (providing false or misleading information),
 (viii.1) section 811 (breach of recognizance),
 (ix) subsection 733.1(1) (failure to comply with probation order),
 (x) paragraph 4(4)(a) of the Controlled Drugs and Substances Act, or
 (xi) paragraph 5(3)(a.1) of the Controlled Drugs and Substances Act.

3) Electable Indictable

 PROCESS CHOICE OF FORUM:


 First appearance PC
 (A) Election  3 options
 (1) Preliminary inquiry in QB if requested
 No preliminary inquiry at PC
 (2) Trial in PC or QB
 (3) If at QB  Chose between Judge and jury or just judge
 S.565(1)(c): If you do not pick you will be deemed to have chosen judge and
jury
 S.(2) – Accused can chose forum
 If an accused is before a justice charged with an indictable offence
other than an offence listed in section 469, and the offence is not one
over which a provincial court judge has an absolute jurisdiction under
553 – the Accused will chose
 S.555 – PC Judge can deem it should be judge and jury
 S.568 – AG can deem should be judge and jury
 (B) Re-election - Accused can change mind and re-elect
 S.561- 563.1 and 565

SUMMARY
PART 27 CODE
 s.785: this part applies to all summary convictions, even those not in the criminal code

Chart 2
 Max penalty, unless otherwise provided is $5,000 or 6 months imprisonment or both
(s.787(1))
 Always tried in PC (Superior Court have jurisdiction but never use it)
 Do not need to appear personally, can ask lawyer or agent to do so

APPLICATION OF THE PART:


 S.786(2) Statute of limitations:
o No proceedings shall be instituted more than six months after the time with the subject
matter of the proceedings arose, unless the prosecutor and defendant so agree
o If not stated in the provision, this is the penalty
o Can be longer than 6 months: i.e. s.266 – allows 18 months for a summary conviction

FORUM:
 Provincial court – QB not available
 S.469: Jury trial not available

PUNISHMENT:
 S.787(1): Unless otherwise provided by law, no more than $5000 fine, 6 months imprisonment
or both
o EXCEPTIONS:
 S.267: Assault with a weapon/causing bodily harm
 Summary or indictable
 If summary can be imprisoned for 18 months
 S.271: Sexual assault
 Summary or indictable
 If summary can be imprisoned for 18 months
 S.785  Definitions

APPEARANCE IN COURT:
 s.800(2): Accused normally need not appear personally in court but may instead choose to have
a lawyer or some other agent appear for him unless the judge orders that he must appear
personally
HYBRID
CHART 4

PENALTY
 If indictable the penalty is usually specified, but if not governed by s.743
 Max penalty when proceeding summarily =18 months
 If indictable, Electable under s.536(2)

PROCESS:
 Chart #4:
o First appearance provincial court
o Election by Crown
 If Crown fails to elect where they normally elect  by default it will be
summary
 Default in CL is always in favour of the accused
 If crown elects summary  Part 27 applies
 Treated as summary – chart #2
 If Crown elects indictable
 Treated as indictable – chart #3

PROCESS – CLASSIFICATION OF OFFENCES


 Relevant sections:
o 469 – 553 – 536(2) CCC

INTERPRETATION ACT
 S.34(1) Where an enactment creates an offence
o (a) the offence is deemed indictable if the enactment provides that the offender may be
prosecuted by indictment;
o (b) the offence is deemed to be summary if there is nothing to indicate that the offence is
indictable; and
o (c) if the offence is hybrid no one will be considered to have been convicted of an
indictable offence if they were convicted of the summary offence
 S.34(2) All provisions of CC relating to indictable offences apply to indictable offence under
other enactments, and all the provisions of the CC relating to summary conviction offences apply
to all other offences created by an enactment, except to the extent that the enactment otherwise
provides.
o If the enactment is silent the code is the one that decides
o If provincial offence  covered by POPA

CHOICE OF FORUM – SHORT FORM


 (1) Provincial court judge
o Quick GP  PC
o Acceptable plea bargain
 (2) Why chose superior court?
o Want complete discovery in Preliminary Inquiry  always a good idea
 PI = disclosure process to establish if there is enough evidence to put the person
on the stand
o Need to delay  stay out of jail
 (3) Judge alone  why would you pick?
o Guilty plea  Choice of judge
o Earlier trial
o Good Defence on the Law
 (4) Judge & Jury
o Sympathetic Circumstances
o Unpredictable (may ignore law)
o Potential for Error Higher
o Good Factual Defence

COUNSEL APPOINTS
Have this discussion with your client; make it clear.

a) Legal counsel – Ultimate decision maker on tactics, strategy and procedure


b) Accused – decision maker on plea – on advice of counsel
 Get your client to make their own pleas so later on they can’t come back and say you
influenced them

ACCUSED

 97% are indeed guilty


 1% are innocent
 2% are guilty of another offence

JURISDICTION OF COURTS
Ell v Alberta 2003 SCR 857
FACTS:
 Dealing with justices of the peace power relating to bail hearings

ISSUES:
 Whether JP are constitutionally required to be independent of the state in the exercise of their
duties. YES

LAW:
 JP are constitutionally required to be independent of the state in the exercise of their duties.

APPLICATION:
 Bail hearings relate to the deprivation of liberty and positive steps have to be taken to keep
infringement on liberty to a minimum
 JPs can also issue search warrants, which if not issued properly, violate s.8 rights
 Respondents exercise judicial discretion
 Otherwise S.7 charter violation

TAKEAWAYS:
 Justices of the peace have jurisdiction under criminal code for certain things:
o 1) Search warrants
o 2) Bail hearings
o 3) Trials of provincial offences

Problem 1 on page 10

1. Frightening the Queen (s.49) – Indictable 469 – 536(4) you can elect a Preliminary Inquiry
2. Murder s.235(1) – Indictable 469
3. Operating vehicle while impaired (s.253, 256) – Hybrid – Crown elects. If the Crown elects
summary there’s no other election – PC trial. Indictment = Accused elects 536(2)
4. Shoplifting (s.334) – Hybrid – Crown Elects – Absolute jurisdiction of PC
5. Robbery (ss.343, 344) – Indictable offence –can elect PC or QB - if QB can elect trial by jury
or judge alone
6. Assault (s.266) – Hybrid – Crown elects summary or indictable
7. Assault causing bodily harm (s.267) – Indictable –Accused elects 536(2) - if QB can elect trial
by jury or judge alone
8. Sexual assault (s.271) - Hybrid – Crown elects summary or indictable
9. Causing a disturbance in a public place (s.175) – summary
10. Possession of stolen property over $5000 (s.355(a)) – indictable
11. Attempted Murder (s.239) – 469 Indictable
12. Conspiracy to commit murder (s.465(1)(a) – 469 Indictable

TIME LIMITATIONS
Summary offence  6 months
 Null and void without consent of accused
 Few exceptions eg: sexual assault

Indictable offence  no time limitations


 One exception under the code
o S.46  Treason  s.48 sets out the limitation

Continuing offence
 Rolling offence that doesn’t occur on a particular event
 EG: Scheme with ongoing fraud
 EG: Kidnapping starts from time of detention and continues until release (R v. Vu)
 Not barred by summary conviction 6 month period
 P.12 Casebook
 11(b) in the Charter

Right to be tried within reasonable time


 Not time limitation for police laying charge, limitation that prosecution must deal with  can’t
lag behind for years

TERRITORIAL LIMITATIONS
S.470Territoriality within Canada
Subject to this Act, every superior court of criminal jurisdiction and every court of criminal
jurisdiction that has power to try an indictable offence is competent to try an accused for that offence
 (a) if the accused is found, is arrested or is in custody within the territorial jurisdiction of the
court; or
 (b) if the accused has been ordered to be tried by
o (i) that court, or
o (ii) any other court, the jurisdiction of which has by lawful authority been
transferred to that court.

S.478 CCC Jurisdiction between provinces


 Court cannot try person who has committed an offence entirely in another province
 Can try an offence that was committed partially in one province and partially in another

S.476 CCC Offence on boundary or where initiated or commenced in another province can be tried
in another
S.6(2) of CCC Can’t be charged with an offence committed outside of Canada GENERAL
PRINCIPLE
o Provides for jurisdiction in the national context
o Exceptions: Terrorist, child sex tourism hostage taking, etc.

R v Greco 2001 ONCA Probation Order valid in Cuba


FACTS:
 Probation order issued in Ontario  had condition keep peace and be of good behavior
 Violent assault on partner in Cuba  argues no jurisdiction

ISSUES:
 Did he have to comply with probation order outside Canada? YES
 Did Ontario court have jurisdiction to try offence? YES

LAW:
 Def Extraterritoriality
o Sovereignty prohibits extraterritorial application of domestic law without consent of the
other state
 Def Territoriality
o Prescribes effects within its own legal system to events taking place abroad
 TEST: Jurisdiction: LIBMAN
o All that is necessary is that a significant part of the activities took place in Canada
o Real and substantial link

APPLICATION:
 (1) Probation order having effect:
o S.733.1(2)  deals only with venue not reach of probation orders
o Policy: Serious and immediate impact within Canada
 Safety of Canadian victim
o Probation order need not have express provision that it applies outside
 No presumption that an order applies only to conduct in Canada
 Common sense approach
 These limitations on ability of court should not be confused with jurisdiction of
court to bind conduct through probation order both home and abroad
 State can extend application of laws outside territory without offending
principles of territoriality
 (2) Jurisdiction:
o Cuba was not interested / did not have the power to enforce terms of probation order
o Real and substantial link = Canada alone had interest in bringing him to justice, not Cuba

TAKEAWAYS:
 (1) S.6(2) of code provides no conviction for criminal offence that takes place outside
Canada
 (2) CC provides for exceptions to s.6(2):
o Aircraft offense, internationally protected persons and things, hostage taking,
nuclear material, war crimes terrorist offences etc. P.13 casebook
 (3) CL in Grego allows that there can be Jurisdiction in Canada for an offence which
occurred outside of Canada based on the principle of territoriality based on R&S link test

Bigelow v R 1982 ONCA Kidnap Case – Any element of crime = Jurisdiction


FACTS:
 Husband got parenting time for the weekend, got on plane to AB and kept the kid
 Tried to argue that because the offence of keeping the kid happened in Alberta there was no
jurisdiction in ON

ISSUES:
 Whether ON court have jurisdiction to hear charge of detaining an infant with intent to deprive
mother of her lawful custody when the child had been taken to AB by father. YES
 Whether anything the appellant did or omitted to do in ON can be considered legally to be an
intrinsic element of the offence. YES

LAW:
 (1) ON courts would have no jurisdiction if entire offence was committed in AB, but if committed
partially in ON have jurisdiction.
 (2) S.432(b) CCC Where an offence was commenced within one territorial division and completed
within another, the offence shall be deemed to have been committed in any of the territorial
divisions.
 (3) TEST: whether any element of the offence has occurred in the province claiming
jurisdiction
o Look to the nature of the offence and the nature of the acts of the accused
o LOOK TO:
 1) Continuity of operation
 Begins in one province and finished in another
 2) Commission of an overt act
 Was any element of offence generated in one jurisdiction 
demonstrable part of the offence
 Look at the act and the nature of the offence
 3) Generation of effects
 Act in one province has effects in another province

APPLICATION:
 (1) Act started in Ontario – continuity of operation
o Keeping the child in Alberta was not the crux of the offence, it was the intentional
withholding
o Had intention when bordering plane in Ontario
o Meaning of word detain  withhold  the withholding effect was in Ontario
o Look at nature and act of the offence
 Nature of offence = interference with the custodial rights of the mother and
child
 Act – is anything that happened in ON a part of the offence? Specific intent to
deny custodial rights happened in ON
 (2) Generation of effects
o Ramifications in Ontario  effects on mother
o Effect is Ontario  mother deprived
 (3) Policy implications  obvious

TAKEAWAY:
 (1) Offence can take place partly in one province and in another
 (2) If so both provinces have jurisdiction
 (3) Bigelow provides test for determining whether court will have jurisdiction were one part has
taken part in one province and one part took place in another province.

JURISDICTION UNDER THE CHARTER


S.24  divided into two parts
 (1) Anyone whose rights and freedoms have been infringed may apply to court of competent
jurisdiction for remedies
o Talks about obtaining remedy with respect to violation of constitutional rights
o Court of competent jurisdiction
 Cannot make charter application in preliminary inquiry court
 Preliminary inquiry court is not court of competent J
 Provincial court is only competent in trial level
o R v Mills p.31
 All criminal trial courts are courts of competent jurisdiction
 Judge in preliminary inquiry is not court of competent jurisdiction
 (2) Excluding evidence
o Evidence SHALL be excluded when admitting would bring administration of justice into
disrepute
o Constitutional law is greater than all other law  any law before 1982 that says
evidence could go in is overruled now

 S.24 preliminary inquiry courts  NO


 S.24 is part of the supreme law of the charter, all law before that would have allowed
evidence to be admitted is subject
 S.52  constitutional law is supreme law any unconstitutional law will fail
SEARCH AND SEIZURE

1. POWERS TO SEARCH AND SEIZE


A search will be reasonable when (Collins, caslake, dedman):
1. it is authorized by a law
2. the law itself must be reasonable; and
3. it is carried out in a reasonable manner

Authorized by Law

a. Statutory
i. There are three ways in which a search can fail to meet the requirement of authorized by law.

• The state authority conducting the search must be able to point to a specific statute or common law
rule that authorizes the search.

• The search must be carried out in accordance with the procedural and substantive requirements
the law provides.

• A search must not exceed its scope as to area and as to the items for which the law has granted the
authority to search (caslake) .

b. Common Law
i. Incident to a lawful arrest (Does not require a warrant nor independent RPGs [Caslake])

• Arrest must be lawful (caslake)

• It must be “incidental to the arrest”


"Truly incidental" to the arrest means that the police must be attempting to achieve some valid
purpose connected to the arrest (caslake).

o Subjective: the police must have one of the purposes for a valid search incident to arrest in mind
when the search is conducted.

o Objective: the officer's belief that this purpose will be served by the search must be a reasonable
one.

o The test is NOT RGS: There must be some reasonable basis [valid objective] for doing what the
police officer did (caslake).

The manner of the search must be reasonable (Stillman, Caslake [citing Belnavis]: Cannot be
conducted in an abusive manner; an arrest for an outstanding traffic fines did not authorize the
search of the trunk of a vehicle, stating "[t]he authority to search as an incident of the arrest does not
extend to searches undertaken for purposes which have no connection to the reason for the arrest.")

There are 3 main purposes for a SILA:

• Ensuring the safety of the police and public;

• The protection of evidence from destruction at the hands of the arrestee or others, and;

• The discovery of evidence which can be used at arrestee’s trial (caslake)


ii. Police Powers
• It has been held that at common law the principal duties of police officers are the preservation of
the peace, the prevention of crime, and the protection of life and property, from which is derived the
duty to control traffic on the public roads (dedman).

• Waterfield test [existence of police powers at c.l.] It is then relevant to consider whether:

1. Such conduct falls within the general scope of any duty imposed by statute or recognized at
common law; and

2. Whether such conduct, albeit within the general scope of such a duty, involved an unjustifiable use
of powers associated with the duty.

Strip Searches (Golden)

1. Arrest must be lawful

2. Search must be ILA

3. Search must be related to the reasons for the arrest itself

4. A strip search will always be unreasonable if it is carried out abusively or for the purpose of
humiliating or punishing the arrestee.

5. Yet a "routine" strip search carried out in good faith and without violence will also violate s. 8
where there is no compelling reason for performing a strip search in the circumstances of the arrest.

6. Such searches are only constitutionally valid at common law where they are conducted as an
incident to a lawful arrest for the purpose of discovering weapons in the detainee's possession or
evidence related to the reason for the arrest. In addition, the police must establish reasonable and
probable grounds justifying the strip search in addition to reasonable and probable grounds
justifying the arrest. Where these preconditions to conducting a strip search incident to arrest are
met, it is also necessary that the strip search be conducted in a manner that does not infringe s. 8 of
the Charter.

7. Strip searches conducted in the field could only be justified where there is a demonstrated
necessity and urgency to search for weapons or objects that could be used to threaten the safety of
the accused, the arresting officers or other individuals. The police would also have to show why it
would have been unsafe to wait and conduct the strip search at the police station rather than in the
field. Strip searches conducted in the field represent a much greater invasion of privacy and pose a
greater threat to the detainee's bodily integrity and, for this reason, field strip searches can only be
justified in exigent circumstances.

Perimeter Searches (kokesch) - Warrantless searches are prima facie unreasonable; burden rests
on party attempting to establish reasonableness. Burden is balance of probabilities. A search will be
reasonable if authorized by law; the law itself is reasonable; and if the manner in which it was carried
out was reasonable. [Collins])

Officer did not have reasonable and probable grounds sufficient to obtain a search warrant; police
lacked statutory authority to conduct the perimeter search.

Is a "perimeter search" carried out under lawful authority pursuant to the common law powers of the
police? No - the common law rights of the property holder to be free of police intrusion can be
restricted only by powers granted in clear statutory language.
Consent

A person should not be prevented from invoking a lack of statutory or common law authority for a
police demand or direction by reason of compliance with it in the absence of a clear indication from
the police officer that the person is free to refuse to comply (Dedman).

In my opinion, the application of the waiver doctrine to situations where it is said that a person has
consented to what would otherwise be an unauthorized search or seizure requires that the Crown
establish on the balance of probabilities that:

(i) there was a consent, express or implied;

(ii) the giver of the consent had the authority to give the consent in question;

(iii) the consent was voluntary in the sense that that word is used in Goldman, supra, and was not the
product of police oppression, coercion or other external conduct which negated the freedom to
choose whether or not to allow the police to pursue the course of conduct requested;

(iv) the giver of the consent was aware of the nature of the police conduct to which he or she was
being asked to consent;

**(v) the giver of the consent was aware of his or her right to refuse to permit the police to engage in
the conduct requested; and,

(vi) the giver of the consent was aware of the potential consequences of giving the consent (Wills).

A warrantless search is prima facie unreasonable. Crown must show complete compliance with
statute or common law to show search reasonable and therefore evidence admissible (Hunter, Rao,
Golden)

Once the accused has demonstrated that the search was warrantless, the Crown has the burden of
showing that the search was, on the balance of probabilities, reasonable. (Caslake)

Search with warrant is prima facie reasonable and onus on defense on a balance of probabilities to
show warrant deficient (Hunter)

CHARTER S.8 + HISTORICAL BACKGROUND


Pre 1982 – Charter
 All relevant evidence was admissible
o Core principle of evidence law
 Before 1982 there was no remedy in criminal law to exclude evidence that was relevant
o If they violated someone’s rights
o Breached law of search at CL
o 1 exception – if the conduct was shocking  extremely rare  in this case evidence
could be excluded
 Accused could possibly afterwards sue the police / file complaint against the police due to their
conduct  this wasn’t often done
 All unlawful searches was not a problem for the crown  most evidence could just go in

Post 1982 Charter comes into force


 Change in law  RE: search and seizure
o s.8 “Everyone has the right to be secure against unreasonable search or seizure”.
 All search/right to counsel law has to be screened through the Charter
o Existing law was constitutionalized  bringing CL up to constitutional standards for
admissibility
 Prior to charter:
o 1) There was common law power to search
 Search incident to arrest (SIA)
 Police allowed to pat people down for things on arrest
o 2) Statutory power to search without warrant
 Provisions in criminal code that police can search statutorily without warrant in
5-6 instances
o 3) Warrant search under CC  statutory power
 All of these pre charter powers to search had to be constitutionalized under the charter
o Being done through s.7-10, s.24 and s.52 of the charter

 Balance between right of a citizen to be free from state intrusion and the need of the state to
intervene on occasion for some legitimate purpose

(A) CRIMINAL CODE

(I) WITHOUT A WARRANT


CRIMINAL CODE
 S.117.02(1) Where peace officer believes on Reasonable Grounds a weapon was used and
it is not practical to get search warrant can search and seize
 S.117.04(2) Where PO has RG to believe someone having weapon is dangerous and they have
ground for search warrant can search and seize without
 S.199(2) Common gaming house
 S.254(2)-(4) Impaired drivers
 S.339(3) Suspected stolen timber
 S.447(2) Cockpits
 S.462 Counterfeit money

(II) WITH A SEARCH WARRANT


CRIMINAL CODE SS.487-489, S.29
 S.487-489  justice (J of Peace or PC Judge) is empowered to issue search warrant in relation to
offence under CCC or other federal statute
 S. 487:Justice need be satisfied there are RG to believe in building receptacle or place:
o (a) Act breaching criminal code has been or is suspected to have been committed
o (b) RG to believe will give physical evidence or whereabouts of person
o (c) RG to believe will be used to committing offence – can arrest the person w/out
warrant
 S 487.1: Tele-warrant Impractical to obtain warrant
 S 487.11: Emergency circumstance for a police Impractical to obtain a warrant
 s. 488: warrants under s 487 must be executed by day, unless it is provided for that it can be by
night

 S 489: Seizure
o Can seize thing mentioned in the warrant, in addition that the person believe on
reasonable grounds that the thing seized
 (a) has been obtained by the commission of an offence
 (b) has been used in a commission of an offence,
 (c) will afford evidence in respect of an offence
o Plain view can be seized, but going into the sock drawer when warrant is for a large safe,
it would appear unreasonable.
 S.29(1): It is the duty of everyone who executes a process or warrant to have it with him, where
it is feasible to do so, and to produce it when requested to do so.

Laporte v Laganiere QUE QB 1972 Removed Bullets – No jurisdiction for warrant


FACTS:
 Search warrant issued to get bullets out of Laporte’s body to see if it matched bullets discharged
by police a year before in a hold-up  was arrested at this time on another unrelated matter
 Unclear whether it could cause harm  if it was going to the search warrant would be quashed
 He did not consent to having it removed

ISSUES:
 Whether the Justice had the jurisdiction to issue the search warrant. NO

LAW:
 (1) Search warrants can be granted to search within a building receptacle or place, and the
human body cannot be viewed as any of these things.
 (2) Have to balance the needs of society and individual rights; if these are in conflict have to
resolve in a way that is consistent with human dignity.
 (3) US law may allow, but not in Canada.

APPLICATION:
 The word “place” in R. 41 of the Federal Criminal Rules of Practice, which allow a warrant to be
issued for a person or place, was “never intended to extend to or include the interior of a human
body”
 If the Crown cannot prove its case against Laporte without doing physical violence to his person
then it is better the case not be proved

NOTES:
 S.256 CCC allows search warrant for a blood sample where caused bodily harm through impaired
driving and unable to consent to breath sample
 Can order DNA orders for serious charges in some cases
 S.487(1) 3 broad requirements of search warrant:
o 1) Informant must provide Justice with information with sufficient factual details to
confer jurisdiction
o 2) Justice must act judicially in assessing facts and exercising discretion
o 3) Warrant must contain sufficient description of the objects of search in relation to
category and offence

Re: Gillis and R1982 Que SC Fishing Expedition – Can’t search to look for evidence
FACTS:
 Fraud case
 Information sworn by officer to get search warrant refers to Annex A
 Information sworn and Annex A have contradictory information

ISSUES:
 Whether the search warrant was within the jurisdiction of the JP and therefore valid. NO
o Reasonable grounds couldn’t have been established based on the information and
annex
o Justice did not adequately describe the objects sought in the search warrant
 What should happened to the seized property. RETURNED

LAW:
 (1) Issuance of search warrant is a judicial act
o Justice must decide whether there are reasonable grounds
o Reasonable grounds is based on:
 1) existence and presence of certain objects in a certain place
 2) their probative value
 (2) P.O. is not judicial officer, cannot decide whether there are reasonable grounds
 (3) TEST: Justice must consider whether the production of the articles will afford evidence
which would be relevant to the issue, and could be used as evidence as fraud (doesn’t have
to result in a conviction, but must afford relevant evidence)
 (4) Purpose of search warrant is to secure things that will be relevant to a case to be
proved, not to secure an opportunity of making observations in respect of the use of
things, and thereby obtain evidence.
 (5) S.443 sets out procedure
o Works with s.8 to protect against unreasonable searches and seizures
o Must be able to identify crime underlying the search OR be able to reasonably
identify what is being looked for
 (6) Statement of Reasonable Grounds: Duty of informant to reveal RPG varies depending on
particular circumstances  If JP can infer the connection or if its obvious it may not be not
necessary
 (7) Description of objects:
o Has to be sufficiently precise with regards to category and relation to the offence for
which they are to provide evidence
 Could have given the date of the items, other identifying numbers, etc.
o Whoever conducts the search has to be able to upon reading the warrant know what is
being sought and what may be sized
 “Where the connection between items sought and the offence is … gathered easily by inference…
the informant is not obliged to underline the obvious” (p46)
 “It is not sufficient for the justice to either rely solely on the good faith of the informant”
 S.8 + S.24  anything unlawfully seized has to be returned

APPLICATION:
 The grounds set out are insufficient
 Only sets out financial statements  needed more specifics about date of offence etc.
 Only the nature of the documents sought is set out but there is nothing to identify them or tie
them with the offence of which they afford evidence
 Have to return the items

R v M (N.N.) 2007 ONCA


Principle requirements for search warrant application:
1. Disclose reasonable grounds or probability regarding the essential statutory pre-reqs
not BRD but more than mere suspicion
2. Key elements to establish credibility-based probability
o Information must set out reasonable grounds
 for believing offence has been committed; and
 that things searched for will afford evidence; and
 that the things in question will be found at specific location
o Disclose a substantial basis for existence of the belief
o Disclose the reliabilityof police informant information.
 Assessed in light of the totality of circumstances. Look at: degree of detail
in hearsay relied on, source of info, indicia of reliability
 It is an ex parte application, so full disclosure is required
 Have to disclose unconstitutional/fraudulent means to obtain to prevent the state from
benefitting from the illegal acts of P.O. without being forced to sacrificed search warrants which
would have been issued in any event.
 TEST for misconduct: whether on the totality of the circumstances the conduct of the police in
seeking prior authorization was so subversive of process that the resulting warrant must be set
aside to protect the process and the preventative function it serves.

R v Hunter ONCA 1987


ISSUES:
 Whether the identity of a police informant should be preserved if it means the validity of a search
warrant cannot be determined. NO

LAW:
 The identity of the informant is part of determining whether there were reasonable grounds
 If producing the informant would bring justice into disrepute, Crown can withdraw the charges
or go ahead on the basis of a warrantless search.
 Trial judge would also have to consider s.24(2) and whether the admission of evidence would
bring the admin of justice into disrepute

Criminal Code Amendment: Anti-gang Legislation, 1997


s.487.3: A judge or justice may prohibit access to and the disclosure of any information relating to
the warrant on the grounds that the ends of justice would be subverted if the information disclosed
 compromised the identity of a confidential informant;
 compromised an ongoing investigation;
 endangers a person involved in intelligence gathering and thereby prejudices future
investigations
 prejudice the interests of an innocent person
 or any other sufficient reason

LAW REFORM COMMISSION OF CANADA WORKING PAPER 30


 Large problems in lots of cities with legality of search warrants
 Gap between the legal reasons for issuing search warrants and the daily realities of practice
o 39.4% of warrants were validly issued
o Usually for reasons that went beyond formalities
o In most cases warrants could have been judicially issued but failed in adherence to
procedure
 Problems more than formal  substantive
o Judicially of warrant issuance  test is the presentation of reasonable grounds to
believe
 Problem resides with adherence to procedures
 Different cities have different procedures  Vancouver is good MTL is bad
 Different local attitudes towards search warrant issuance  some view as formality

(B) COMMON LAW


INTRODUCTION
3 TYPES OF COMMON LAW SEARCH:
(1) INCIDENT TO ARREST
(2) CONSENT SEARCH
(3) ANCILLIARY POWERS

(I) SEARCH INCIDENT TO ARREST


INTRODUCTION
PRE-CHARTER:
 SIA included: (Brezack)
o Mouth
o Pat down search  over clothes
o Car nearby
o Body cavity (Renyan)
 Common law search incident to arrest (SIA) included intrusive searches with high expectation of
privacy  (Renyan)
o Before charter all evidence that was relevant was admissible
 Only limitation on what police could do incidental to arrest was no surgical procedures (Laporte)

POST-CHARTER:

(1) Arrest must be lawful


 R v Storrey
 Has to be reasonable probable grounds for arrest  objective test
 If police fail here the whole search fails
 Did the police have RPG to arrest the person  if argument can be made by defense council that
they couldn’t search is lost

(2) Search must be incident to the arrest


 R v Tomaso: must have reasonable probable grounds to arrest for the search to be lawful
 R v Stillman: 3 requirements for a search incident to arrest
1. A search must be reasonable;
2. It must be authorized by law; and
3. The manner in which the search is carried must be reasonable
 Has to be related to / connected to the reason for the arrest
 Caslake
 Stillman
o Arrested for traffic tickets  without true consent they searched the vehicle
o Would this be related to the reasons for arrest (arrested him for traffic tickets) search of
the trunk not connect to reasons for arrest

(4) Search must be carried out reasonably once SIA has been found

WHEN IS SEARCH AUTOMATIC AND UNQUALIFIED UNDER SIA?


 1) Pat down search  need only one RPG needed for any crime
o The main purpose is to make sure no weapon
o This is completely unqualified
o Need only RPG for arrest, no need for RPG that the person has something illegal
 2) Site search  allowed  Stillman shows
o Based on lawful arrest
o Not clear if this is qualified in some way
o Does it mean within a block
 3) Car search
o Stillman  car can be searched as an aspect of SIA
o Is it unqualified  NO
o Caslake – it is qualified temporally with respect to cars
 Car has to be qualified
 SIA has qualification of what they’re searching for, has to be incidental to arrest
 4) Mouth search
o Breszak  mouth search was fine as SIA
o Now that charter is in  not talked about specifically in Stillman
o Collins puts on parameters for mouth searches
o Mouth search is not a body cavity search legally  separate
 5) Body cavity search
o Nothing specifically said in Stillman
o Golden case  can have body cavity search as incident to arrest, however it is highly
qualified by Golden
 6) Body samples
o Prohibited under SIA

R v Brezack ONCA 1949


FACTS:
 Accused was known to PO has having been charged for drug possession before
 Saw another drug addict approached him  believed accused had drugs
 Tried to force finger into accused’s mouth to prevent him from swallowing any drugs that might
be in his mouth
 While forcing finger in mouth, accused bites PO  PO wrestle him to the ground with
considerable force, accused continues to bite and kick
 No drugs found in mouth or on person
 Drugs found in his car around the corner
 Charged with assault PO and possession

ISSUES:
 Whether the search was part of the lawful execution of the PO’s duty making arrest  whether
the search was incidental to the arrest. YES

LAW:
 For a warrantless search to be reasonable, it has to be incidental to the arrest, and be based on a
reasonable belief on the part of the PO that the search is necessary for the arrest.
 Mouth search is justifiable as an SIA at common law (qualified by Collins: mouth search is
permissible if it’s a known drug dealer, not otherwise)

APPLICATION:
 PO putting fingers in mouth was part of reasonable efforts  had reason to believe he had illegal
drugs
 Finding the drugs in the car supports the reasonableness of the PO’s belief he had drugs in his
mouth
 Search was reasonable as incidental to the arrest – the circumstances justified the search a
reasonable precaution
 As soon as they grabbed him he was under arrest
Laporte v Laganiere 1972 QU QB
ISSUES:
 Whether the power to search as incident to an arrest includes a surgical operation. NO

LAW:
 SIA: limited to that search which is incidental to the making of an arrest or the continued
detention of the prisoner in safe custody.
o Reasons: make arrest effective, ensure that evidence does not disappear and to prevent
the commission of a further offence
 Does not justify surgical intrusion into the body of a prisoner.

R v Tomaso ONCA 1989


FACTS:
 Accused involved in fatal car accident  was unconscious at hospital
 P.O. took sample of blood coming out of his ear while he was unconscious
 Two weeks later the blood report came back  was impaired at the time

ISSUES:
 When they took the blood from his ear did they have RPG to arrest? YES
 Whether the taking of the blood from the ear was authorized as a lawful search incidental to
arrest despite the fact that it was done prior to the arrest. NO
 Did the P.O. have reasonable grounds for the arrest prior to taking the sample of the blood? YES

LAW:
 A search preceding arrest may be incident to arrest provided that the P.O. had RG, prior to the
search, for arresting the accused.
 S.245(3)  allows for taking of blood where officer has RPG that it will not be possible to get
breath samples
 S.256  allows the issue of a warrant for blood samples where there is an accident resulting in
death or bodily injury to any person, a medical practitioner says consent is not possible and is of
the opinion that it would not endanger their life to provide a sample.

APPLICATION:
 This accident in this case occurred before the law above was passed, so the search was not
lawful.
 If the current law was applied, the court could have found that the P.O. had reasonable grounds
that the accused was impaired before taking the samples of blood  the analysis of the blood
samples was part of the justification for such arrest
 Despite being before the arrest, it was incident to the arrest because he had reasonable grounds
for the arrest prior to taking the samples.

TAKE AWAY:
 (1) There has to be RPG for the arrest for SIA to be lawful
 (2) Police can make a seizure prior to the arrest so long that they can establish after the
fact that they had RPG for the arrest
o Search can take place before the arrest so long as there are RPG for arrest

R v Stillman SCC 1987


FACTS:
 Group of teenagers partying, 17 year old boy leaves with 14 year old girl  girl is found dead
with head wounds, bite marks and was raped
 Boy turns up wet from waist down, cut above one eye, scratches, mud and grass on pants
 Boy is taken into police station, refuses searches without lawyers  police take hair, pubic hair
and saliva by force  boy is released
 2 weeks later after DNA comes back, forcibly take dental molds without consent
 While in custody, boy blows his nose and throws the tissue in the garbage
 Everything comes back as matching

ISSUES:
 Whether the taking of the hair, teeth and saliva samples were authorized as being search
ancillary to arrest? NO
 Whether taking the tissue from the garbage was deemed violate s.8? YES  but didn’t pass
Grant analysis
 Whether SIA authorized the seizure of bodily substances. NO

LAW:
 (1) 3 requirements for search to be found reasonable:
o 1) authorized by law
 Either statutorily authorized (in the code or by warrant)
 At common law  search incidental to arrest
 If appellant has no reasonable expectation of privacy
o 2) law itself must be reasonable
o 3) manner in which the search was carried out must be reasonable
 (2) 3 conditions for search to be authorized as incidental to arrest
o 1) Arrest must be lawful
o 2) Search must have been conducted as incident to the arrest
o 3) manner in which the search was carried out must be reasonable
 (3) Warrantless search is presumed to be unreasonable  SIA is exception to this general rule
o Rule from Hunter v Southam
 (4) Purpose of SIA is to:
o Prevent escape of arrested person
o Allow P.O. to search for weapons
o Prevent evidence under the control of the detainee from being destroyed
 (5) Limits on SIA:
o Discretionary  police can exercise discretion when not to search
o Has to be for a valid objective in pursuit of the ends of criminal justice
o Search must not be conducted in an abusive fashion
 (6) SIA cannot authorize the taking of bodily samples – departure from CL
 (7) Discarded things:
o No reasonable expectation of privacy when a thing is discarded/abandoned  when
you’re in custody, you’re not abandoning things even if you throw them in the garbage
because you have no choice where to put them

APPLICATION:
 Taking of hair/dental impressions is highly intrusive  also there is no likelihood that his DNA
or dental impressions will change  doesn’t fall within the purpose of SIA
 The tissue was abandoned in a garbage can, but because he was in custody he had no other
option of where to put it  since he said that he did not consent to any bodily fluids being taken,
this is a breach of s.8
o Did not pass Grant analysis
o McLachlin writing for the minority said the tissue was abandoned and fair game

TAKE AWAYS:
 (1) Stillman allows SIA as having been constitutionalized under the Charter
 (2) Criteria for all searches is set out in Stillman  all, not just SIA  RE: Collins case
 (3) 3 requirements for SIA to be lawful
 (4) SI is not unlimited  does not permit the taking of body samples
 (5) SIA includes evidence at the site? NOT CLEAR
o Includes nearby motor vehicles not clear temporally
 (6) IF there are RPG for arrest that is all the RPG that are required  P.O. doesn’t have to have
grounds to believe after arresting the person that they believe there is anything to search for 
just need one RPG to do a pat down search
o Even traffic ticket situation can pat down  you could have weapon
 (7) Stillman leaves unsettled the following areas:
o Strip search
o Mouth search
o Body cavity searches

R v Caslake 1998 SCC


FACTS:
 Accused was seen in a field where a garbage bag of weed was found
 Accused was arrested in his car, which was parked nearby
 The car was searched after it was impounded, pursuant to police impound policy
 Search of the car found $1400 and 2 x ¼ g bags of blow

ISSUES:
 Whether the search of the car was incidental to the arrest and therefore authorized under
common law? NO – Majority found violates s.8
 Whether the evidence should be excluded for temporal reasons. NO
 Whether the evidence should be excluded? NO – doesn’t pass Grant

LAW:
 Limits on SIA are derived from its justification  must be truly incidental to arrest 
search is only justifiable its purpose is related to the purpose of the arrest
 Truly incidental means some valid purpose connected to the arrest  answers question:
what were the PO looking for and why?
 Subjective element  PO has to turn mind to purpose incidental to the arrest
 Objective element  the PO’s belief that he had legitimate reason to search was
reasonable in the circumstances
 Not RPG  just need to have reasonable basis for action + valid objective of search
 What is the purpose of SIA?
o (1) Officer safety
 Have to be able to pat person down to make sure that they don’t get stabbed or
shot
 Police obligation
 If don’t pat down will be charged and disciplined under the police act because
causes risk to officer
o (2) Preserve evidence
 Preserve evidence for any crime
 If PO has grounds to arrest officer for theft, when they’re patting that
person down if they find drugs or counterfit money or anything else
that is illegal they can lawfully seize that  not just for the thing that
the officer approached that person to arrest them for  for anything
illegal
o Wide context to get evidence of ANY CRIME
o (3) Discovering evidence for trial

APPLICATION:
 (1) Had the objective justification at search when arrested for trafficking, but the search was
actually conducted due to a subjective desire to comply with policy  not reasonable  did not
turn mind to incidental purpose of search when searching
 (2) The search of the car was to do an inventory of the car NOT for a purpose that was actually
incidental to the arrest
o Did they search it for reason connected to the arrest? NO
o Reasons for the arrest were drugs  if they’d gone to search the car for something
related to drugs it would have been fine, but was not the case, went for something else

DISSENT:
 No onus on Crown to prove subjective element  no subjective element

TAKE AWAY:
 1) Search of car as lawful SIA is confirmed – still man set out but no parameters
 2) Police do not have to have additional RPG to search the vehicle (same as pat down  just the
RPG from the arrest)
 3) Must be truly incident to the arrest  connected to or related to the reasons for the arrest –
an inventory search doesn’t qualify
 4) Temporal context of the search can either allow at the SIA can either allow that the search is
lawful or not lawful
o When the search occurred

(II) ON CONSENT
LAW REFORM COMMISSION OF CANADA WORKING PAPER 30
 Theory of consent search is founded on proposition that the search performed does not
constitute an actionable intrusion
 Doctrine of volenti non fit injuria: one who has invited or assented to an act being done towards
him cannot complain of a wrong
 Consent = waiving right to invoke normal legal protections  take out of public law realm and
makes it a private interaction between individuals
 Rarely litigated cases
 Raises issues around true, limited and informed consent
CLASS
 If there is true voluntary consent and no consequences for refusing there is no charter issue
 In every instance that they can, police will say it was a consent search  Crown will argue there
is no charter issue

Dedman v R SCC 1985


RATIO:
 Submission to a PO’s exercise of apparent authority cannot be characterized as voluntary or
consensual unless it was clear to the person at the time that he was free to refuse to comply.
FACTS:
 Pulled over voluntarily after being signaled to do so at a police roadblock

ISSUES:
 Whether complying with police instructions removes the requirement that a PO have a statutory
or common law authority to be exercising power. NO

LAW:
 Consent to a PO pulling you over does not absolve the PO of needing lawful statutory or CL
authority for the stop, because unlike with a civilian, you are not assured that you will have
impunity for not complying.
 PO only act lawfully if they act in authority that is from statute or derived as a matter of CL from
their duties  If PO can’t pin their action to statute or common law they’re outside their
authority

APPLICATION:
 He did not really have the choice not to stop because of the power of the police, therefore the
police needed a lawful basis to pull him over, which they did not have

TAKEAWAY:
 (1) PO must ground authority in statute or CL  if act outside of this they act unlawfully
 (2) No true consent unless a person is free to refused to comply without consequences

2 ONCA CASES: PO HAVE TO INFORM OF RIGHT NOT TO CONSENT 


SCC HAS NOT CONFIRMED

R v Wills ONCA 1992


LAW:
 Waiver doctrine: where someone has consented to what otherwise would have been an
unauthorized search or seizure requires the Crown to prove on BoP:
o 1) There was consent
o 2) Giver of consent had the authority to give the consent in question
o 3) The consent was voluntary and not coerced
o 4) Giver of consent was aware of the nature of the police conduct to which he/she
was being asked to consent
o 5) Giver was aware of right refuse consent
o 6) Giver was aware potential consequences of giving consent

R v Lewis ONCA 1998


LAW:
 PO not under duty to advise person of right to refuse consent under s.8 doesn’t have
informational component like s.10(b)
 If not advised of right to refuse could lead to violation of s.8 where police conduct can be justified
only on the basis of INFORMED consent
o In that case, up to TJ to determine that any apparent consent given is not consent at all
for the purposes of s.8

(III) INCIDENT TO DUTIES OF POLICE OFFICERS –


ANCILLARY POWERS DOCTRINE
INTRODUCTION
 Grounding case is Deadman #2  majority decision in particular and part of Dickson’s dissent
 Principles of doctrine  set out carefully in dedman 2
o Concepts of duty and power
o When PO has a duty do they have a power to act on that duty  the answer is not
invariably
o Duty might = power, but duty might not = power
o When will duty = power? When it is reasonably justifiable and reasonably necessary
NEED TO KNOW CL DUTIES AND STATUTORY DUTIES BECAUSE THESE DUTIES CAN = Power
 Alberta Police Act
o 38(1) Every police officer is a PO and has the authority, responsibility and duty
 To perform all duties that are necessary
 to carry out the PO functions as a PO  CL
 3 duties  set out in Dedman #2
 Prevention of crime
 Encourage and foster relationship
 Apprehend persons who may lawfully be taken into custody
 Execute all warrants

Colet v R SCC 1981


RATIO:
 Any provision authorizing police officers to search and enter private property must be phrased
in express terms and the provisions of the Interpretation Act are not to be considered as clothing
police officers by implication with authority to search where s.105(1) and the warrant issued
pursuant thereto are limited to seizure.

FACTS:
 Municipal public works directed by city council to clean up accused’s property, including tearing
down the shack he lived in illegally
 Accused made clear he would not allow them to come onto his property  RCMP believed he
might keep them off the property with guns
 RCMP obtained a warrant pursuant to s.105(1) which allows RCMP to seize guns when they have
to for RG
o Warrant only gave power to seize not to search
 Got on roof of house, said was warrantless search and threw gasoline on the POs

ISSUES:
 Whether s.105(1) can be read to include the power to both search AND seize even though the
legislation only reads seize. NO
 Whether the property rights of the individual can be invaded otherwise than with specific
statutory authority. NO

LAW:
 Eccles v Bourque  gives power to the police to go into a house without permission to arrest an
individual
o Does not give the power for any reason pursuant to public interest
 Police are not justified in coming into a home until they have announced their presence and
demonstrated their authority by stating a lawful reason for their entry

APPLICATION:
 Since the warrant only gave the police officers power to seize, they had no powers to search and
were trespassers
 Further they didn’t properly demonstrate their authority, as they only waived a warrant from far
away

Dedman v R SCC 1985 (2)


FACTS:
 Accused did not provide breath sample at RIDE checkstop  was acquitted because the police
did not have lawful basis to stop him and therefore he had a reasonable excuse to not provide a
sample
 At this time there was no federal or provincial regulation allowing this kind of checkstop
 This is the appeal

Majority – Le Dain (McIntyre, Lamer, Wilson JJ)


ISSUES:
 Whether despite not having legal authority for the RIDE program, the police stop was authorized
due to common law authority derived from the “powers associated with duty”. YES

LAW:
 WATERFIELD TEST  answers both “whether a PO was acting in the execution of his duties” and
“whether the officer had common law authority for what he did.”
o Considers what the police constable actually did and in particular whether such conduct
was prima facie an unlawful interference with a person’s liberty or property  answer
is YES:
 A) Such conduct falls within the general scope of any duty imposed by statute or
recognized at CL, and
 B) Whether such conduct, albeit within the scope of such a duty, involved an
unjustifiable use of power associated with the duty.
 (i) The interference with liberty must be reasonably necessary for the
carrying out of the particular police duty and it must be reasonable,
having regard to the nature of the liberty interfered with and the
importance of the public purpose served.

APPLICATION:
 (1) Right to circulate on a road in a car  qualified liberty
o Qualified because it is a licensed activity and not fundamental
 (2) Random vehicle stop is PF unlawful interference with liberty as it is not authorized by statute
 (3) Fell within general scope of the duties of a PO to prevent crime and protect life/property by
controlling traffic
 (4) Whether the interference was reasonably necessary?
o Factors:
 Licensed activity
 Stops are arbitrary
 Psychological effects on innocent drivers
 Short duration and slight inconvenience
o Weighing factors  not an unjustifiable use of power  there was common law
authority for the purpose contemplated by the RIDE program
Must let you know that there is a choice to refuse the officer (for legitimate consent).

Dissent – Dickson (Beetz and Chouinard)


ISSUES:
 Whether the police have the power at CL (in other words the lawful authority) to execute their
general duties by means of random stops of motorists when they have no reason to believe that
the motorist has committed or will commit an offence. NO

LAW:
 Absent explicit or implied statutory authority the police must be able to find authority for their
actions at CL, otherwise they act unlawfully
 Police have the power to question but no power to compel an answer before arrest
 Waterfield provides no support for PO conduct that is unlawful  For Waterfield to apply, PO
must be engaged in lawful execution of their duty at the time of the conduct in question

APPLICATION:
 RIDE  no statutory authority
 PO have no authority to detain person for questioning at CL short of arrest  this means they
also have no basis to stop and detain a motorist arrested in the circumstances of this case
 The stopping is entirely arbitrary with no grounds  police not authorized to stop
 Slippery slope should be up to parliament to legislate if police powers are needed
o Police will use these powers in other contexts, and have

CLASS:
 When you make argument that officer had lawful authority have to tie it to duties either
statutory or CL

TAKEAWAYS:
 (1) Common law duties p.93
o 1) preservation of the peace
o 2) prevention of crime
o 3) protection of life and property  traffic
 (2) PO may have power to search arising from officer duty if it is reasonably necessary
and justifiable (p. 95) – e.g. if he thought the building might burn down
 (3) These ancillary powers are not unlimited, see Colet Case. Police have a duty under s.
105 of the code to seize firearms but in those circumstances they did not have the power
 (4) The police can question anyone on anything

(C) OTHER FEDERAL STATUTES


 Contain powers of search without warrant
 Until 1985 Criminal Law Amendment Act allowed for writs of assistance  deemed
unconstitutional  now include a telewarant procedure

CONTROLLED DRUGS AND SUBSTANCES ACT SS.11, 12


 Broad powers of search are bestowed in CDSA

(D) PROVINCIAL STATUTES

(E) CONSTITUTIONAL MINIMUM STANDARDS


(I) WARRANT REQUIRED

Hunter v Southam 1984 SCC


FACTS:
 Southam owned Edmonton Journal
 Hunter was the director of research and investigation for the Combines Investigation Act
 Hunter authorized a search of the Edmonton Journal under S.10(1) and s.10(3) of the Combines
Investigation Act
o These sections required a certificate be provided to the Director first
o The day before the Charter was proclaimed protecting s.8 rights the certificate was
issued
o 2 days later the certificate was provided and the search was executed
o Wanted to search
 Director authorized others to examine documents and other things at the Edmonton Journal
 Southam applied for interim injunction  search violated S.8 of the new Charter
ISSUES:
 Whether the search powers granted by s.10(1) and 10(3) of the Combines Investigation Act
violate s.8 of Charter  unreasonable search and seizure. YES

LAW:
 (1) S.8 rights have to be at minimum what was guaranteed by common law minimum
prerequisites were:
o Power to authorize a search and seizure is given by an impartial and independent
person who is bound to act judicially in discharging that function
o Evidence must satisfy justice the person seeking authority has reasonable ground
to suspect an offence has been committed.
o Evidence must satisfy the justice that the person seeking the authority has
reasonable grounds to believe that something will afford evidence of an offence
may be recovered; and
o That there must be evidence on oath before him.
 (2) Right to be free from unreasonable search and seizure protects only a reasonable
expectation of privacy.
 (3) Gov’t actions should seek prior authorization rather than subsequent validation  s.8 seeks
to prevent unjustified searches
 (4) Prior authorization (eg. Warrant) requires state to demonstrate the superiority of its interest
to that of the individual
 (5) Where it is feasible to obtain prior authorization, such authorization is a precondition for a
valid search and seizure
 (6) Warrantless searches are presumptively unreasonable, and it falls to the state to rebut this
presumption.
 (7) Person who gives prior authorization need not be a judge, but need act judicially  has to be
division between administrative and judicial function
 (8) MINIMUM TEST s.8:
o Reasonable probable grounds, established upon oath, to believe that an offence
has been committed and that there is evidence to be found at the place of the
search constitutes the minimum standard consistent with s.8 of the Charter for
authorizing search and seizure.

APPLICATION:
 (1) Crux is the definition of “unreasonable” in the s.8 guarantee of freedom from unreasonable
search and seizure
 (2) S.8 does not provide authorization for gov’t action but limits government action  limitation
on whatever powers of search and seizure the federal or provincial gov’t already and otherwise
possess
 (3) Right was originally to protect property, now protects property and privacy  this is the
bare minimum of the right
o Guarantees broad and general right to be free from unreasonable search and seizure
 (4) Prior authorization  not enough in this case
o Overlapping of functions  administrative and judicial
o Not impartial person granting the order to search

(II) WARRANTLESS SEARCHES

R v Collins SCC 1987


RATIO: mouth search is permissible if it’s a known drug dealer, not otherwise
FACTS:
 Police watching accused and her husband, had tip that they were drug smugglers
 He went outside  PO found heroin in his car
 Went back to accused, grabber her by throat to keep her from swallowing drugs
 Saw something in her hand, told her to drop it  was balloon with heroin
 Was this a common law search incident to arrest?
o Warrantless search = crown must prove RPG on a BOP (not a SIA case)

ISSUES:
 Whether the PO had reasonable grounds to believe that there was a narcotic in place when they
throat searched the accused, so as to rebut the presumption that the warrantless search was
unreasonable. NO – evidence not led at trial.

LAW:
 (1) Burden of proving a charter breach is on the accused
o If court doesn’t know whether charter was breached, resolves in favour of the
crown
 (2) If there is a warrantless search, the onus is on the accused to prove the search was
warrantless  onus then shifts to the Crown to prove the search was reasonable.
 (3) Search will be authorized by law if it is authorized by law, if the law itself is
reasonable, and if the manner in which the search was carried out is reasonable.
 (4) For search to be lawful Crown must establish that the officer believed on reasonable
grounds that there was something illegal.

APPLICATION:
 The reasonable grounds here (that she was a drug dealer) could also go to the
reasonableness of the search  EG: throat searching her
 Crown did not present evidence with regards to the officer's belief in reasonable grounds
 didn’t discharge duty
 New trial ordered

R v Golden 2001 SCC


FACTS:
 Accused was seen selling drugs – was detained and patted down, no weapons were found
 Observed Accused through a telescope – saw him at a Tim Hortons suspected drug dealing,
Accused was giving out a white substance
 Send 4 POs to arrest. Accused and 2 others were arrested. Found what they thought was crack
cocaine under a table, arrested the Accused, did a pat down but found nothing
 Accused was strip/cavity searched in the Tim Hortons in brutal way  found crack in his butt

ISSUES:
 Whether the strip search violated the accused’s s.8 rights. YES
o Unreasonable
o Violated s.8 as it was in a public place

LAW:
 Warrantless searches are prima facie unreasonable – Crown must prove reasonableness.
 SIA is exception to the rule that warrantless searches are PF unreasonable.
 Strip searches cannot be carried out as a routine policy as they are humiliating and degrading
 For strip search to be SIA needs:
o 1) Arrest has to be lawful
o 2) The strip search has to be truly incidental to the arrest  related to the reasons for
the arrest itself
o 3) carried out in a reasonable manner
 Also needs to be TRULY necessary to prevent the destruction of evidence or the harm to officers
 Mere possibility that an individual may be concealing evidence or weapons is not
sufficient to justify a strip search
 Def of Strip Search: Removal of some or all of the clothing of a person for VISUAL search or
PHYSICAL INSPECTION ADD TO THIS -limited to genitals, buttocks or breasts

 Will always been unreasonable if carried out for the purpose of humiliating or punishing the
detainee
 Will also be unreasonable if carried out in a reasonable way, but with no compelling reason for
performing a strip search in the circumstances of the arrest
 Need:
o RPG for arrest
o RPG that a strip search is necessary
o Strip search has to by truly incidental to the arrest
o Has to be carried out in a manner that does not infringe the Charter
 Strip searches should only be carried out at police station unless there is a demonstrated
necessity and urgency to search for weapons or objects that could be used to threaten the
safety of the accused, the arresting officer or other individuals

APPLICATION:
 Although the arrest was lawful on RPG and the strip search was related to the purpose of the
arrest, but the Crown failed to prove that the strip search was necessary and urgent
 The strip search in the restaurant was also carried out in an unreasonable manner

FRAMEWORK for the Police in deciding how best to conduct a strip search incident to arrest in
compliance with the Charter – PACE Factors
1. Can the strip search be conducted at the police station and, if not, why not?
2. Will the strip search be conducted in a manner that ensures the health and safety of
all involved?
3. Will the strip search be authorized by a PO acting in a supervisory capacity?
4. Has it been ensured that the PO carrying out the strip search is of the same gender as
the individual being searched?
5. Will the # of POs involved in the search be no more than is reasonably necessary in
the circumstances?
6. What’s the minimum of force necessary to conduct the strip search?
7. Will the strip search be carried out in a private area such that no one other than the
individuals engaged in the search can observe the search?
8. Will the strip search be conducted as quickly as possible and in a way that ensures
that the person isn’t completely undressed at any one time?
9. Will the strip search involve only a visual inspection of the arrestee’s genital and anal
areas without any physical contact?
10. If the visual inspection reveals the presence of a weapon or evidence in a body cavity
(not including the mouth), will the detainee be given the option of removing the
object himself or of having the object removed by a trained medical professional?
a. If this is not done, a body cavity search may be found to be unreasonable.
11. Will a proper record be kept of the reasons for and the manner in which the strip
search was conducted?

(III) UNLAWFUL SEARCHES VIOLATE S.8

Kokesh v R 1990 SCC


FACTS:

 RCMP received a tip that K was involved in trafficking marijuana
 Observed K in a ferry to Van Island, followed him, did an aerial observation
 2 days later, 2 RCMP officers at 2am officers conducted a perimeter search. Made a number of
observations, including condensation and humming, detected odour of marijuana and heat
coming from the premises
o No search warrant to go onto the property
o Cst Povarchook said he didn’t have RPG for a warrant, only a suspicion, then increased
from suspicion to solid grounds
 Charged with possession and trafficking
 Got search warrant  went inside and found plants

ISSUES
 Whether the initial warrantless search was reasonable and therefore authorized by law so as to
rebut the presumption that any warrantless search is unreasonable. NO

LAW:
 Hunter v Southam: Warrantless search is presumptively unreasonable and the party seeking to
justify the search has the onus of proving on the balance of probabilities that the search was
reasonable.
 A search will be reasonable if it is authorized by law, if the law itself is reasonable and if the
manner in which the search was carried out is reasonable.
 Common law property rights to be free of police intrusion can be restricted only by powers
granted in clear statutory language.

APPLICATION:
 The PO admitted he did not have RPG to get a search warrant and do a lawful search
 There is no common law or statutory law authorizing the search  means the search was
unreasonable and violated s.8

Class Notes
 “Given the concession by Officer Povarchook at the preliminary hearing that he did not have
reasonable and probable grounds sufficient to obtain a search warrant, the inevitable conclusion
is that the police lacked statutory authority to conduct the perimeter search.”
o Police had the onus of proving the search was reasonable
 Collins TEST: (1) must be authorized by law (common law or statute), (2) if the
law is reasonable, and (3) if the manner it is carried out is reasonable
 Court rejected that it was authorized by statute.
 Crown said it was authorized by the ancillary powers doctrine court said
no, referred to Colet, this is a private residence (Colet: duty does not give
power)
 High expectation of privacy in private homes – Dissent by Dickson in
Deadman, slipper slope argument

(IV) REASONABLE EXPECTATION OF PRIVACY

 R v Wong SCC 1990 – “Whether in a society like ours persons who retire to a hotel room and
closed the door behind them have a reasonable expectation of privacy?”  YES
Airports
 R v Simmons 1988 SCC – 3 types of boarder searches:
o Least intrusive = luggage searches, frisk or pat searches
o Strip or skin searches
o Most intrusive = Body cavity searches – medical doctors, x-rays, etc.
 R v Monney SCC 1999 – compelled urine and stool samples do not violate s.8 or s.9
 R v Buhay 2003 SCC – Reasonable expectation of privacy in a locker rented at a bus station
However….
 R v McKinlay Transport Ltd SCC 1990 – “individuals have different expectations of privacy in
different contexts…the standard of review of what is “reasonable” in a given context must be
flexible if it is to be realistic and meaningful” (140)

R v Edwards SCC 1996


FACTS:
 Accused charged with trafficking cocaine – 2 confidential informants
o Suspected of having drugs on his person, in his car or at his girlfriend’s place
o PO followed him, found he had a suspended license, stopped him and searched him.
o Suspected he swallowed a golf ball sized amount of hashish wrapped in cellophane
o PO searched his GF’s apartment – the drugs were there. NO search warrant.
o She had told police about the drugs and where they were. They arrested her, later
released her
o She was not told that she had right to counsel or not to consent to the search
 He did not live there, but had a key. Argued that he has a reasonable expectation of privacy.

ISSUES:
 Whether s.24(2) provides remedies to third parties. NO
 Whether the accused had a reasonable expectation of privacy in his girlfriend’s apartment. NO
o Was not living there, not paying rent, only helped to buy a couch. Had a set of keys.
 Court: privileged guest. NO reasonable expectation of privacy.

LAW:
 STEP 1: Was there reasonable expectation of privacy
o Privacy rights of third party not relevant
o Extent of invasion of privacy not relevant here
The factors to be considered in assessing the totality of the circumstances may include, but
are not restricted to the following:
o (i) presence at the time of the search;
o (ii) possession or control of the property or place searched;
o (iii) ownership of the property or place;
o (iv) historical use of the property or item;
o (v) the ability to regulate access, including the right to admit or exclude others from the
place;
o (vi) the existence of a subjective expectation of privacy; and
o (vii) the objective reasonableness of the expectation.
 STEP 2: Was the search reasonably conducted
o Privacy rights of third party may be relevant here
 S.8 only protects those places where the Applicant has a reasonable expectation of privacy.
o Reasonable here implies that an assessment must be made as to whether the public’s
interest in being left alone must give way to the gov’t interest in intruding on individual
privacy for law enforcement.
 Conduct of the police isn’t a factor at the REP stage of the analysis; comes in during part 2 of
determining whether the search was reasonably conducted.
 Reasonable expectation of privacy: essential to this is the right to be free from interference 
key element to REP

APPLICATION:
 Did not have REP to property:
o Was just a visitor
o Did not contribute to rent/expenses
o Had key but did not have ability to regulate the premises  could not exclude anyone
 Can’t argue for a remedy because GF’s rights were infringed  can only be the person whose
rights are being infringed who can make the application

Class Notes:
 p144: Read and know these points
1. Must be the Charter rights of the individual claiming a breach
2. Protects people and not places
3. Personal rights to privacy must be violated to challenge legality of search
4. Burden is on the accused
5. Totality of the circumstances are looked at
6. Factors for totality of circumstances
7. Reasonable expectation of privacy is established – proceed to 2nd stage –
was the search conducted in a reasonable manner?
8. Factors that mitigate against reasonable expectation of privacy
a.
 Privacy – key element is the ability to control the premises

R v M(MR) 1998 SCC


FACTS:
 School dance – principle had tip from students that one student was dealing drugs
 Called police – took student into office first and said that he would search him for drugs
 Found drugs in his sock, removed a cellophane bag and gave
o Used the School Act to justify the search, which was not actually allowed by the Act
o The Act provided the right to provide a safe environment and proper discipline.
 Officer can and did the search  found drugs

ISSUES:
 Whether the Charter applies to school officials. YES
 Was the search authorized by law (common or statute)?
o YES based on the School Act even though the legislation didn’t specifically authorize it
 Did the appellant have a reasonable expectation of privacy in the circumstances? NO

LAW:
 (1) REP needs subjective expectation in privacy and this expectation needs to be
objectively reasonable.
 (2) Student’s reasonable expectation of privacy in a school environment is significantly
diminished
 (3) TEST RPG in school context: reasonable grounds to believe an offence has been
committed and that there is evidence to be found at the place of the search
o TEST whether school search is reasonable: school official needs to have
reasonable grounds to believe a school rule has been or is being violated and that
evidence of that violation will be founds in the location or on the person searched
 (4) Summary:
o 1) Warrant not necessary
o 2) Need reasonable grounds
o 3) Reasonable grounds should be assessed through the preferred position of
school authorities
 (5) Factors for whether the search is reasonable in school setting:
o 1) Authorized by Education Act
o 2) Search carried out in reasonable manner
o 3) Consider all the surrounding circumstances

The absence of a warrant in these circumstances will not lead to a presumption that the
search was unreasonable. Wow!!

APPLICATION:
 Search was authorized by education act and carried out in reasonable manner  reasonable
search
 Majority said VP was not agent of the police  seizure happened before the police came
 (1) Does the charter apply to school authorities  yes P.148
 (2) Was VP acting as agent of police  NO: Was cooperating with police but not agent p.148
o Dissent  Major disagrees
 (3) Do youth have REP when they are at school  yes but significantly diminished
 (4) Is it authorized by law?
o Goes to Education Act to provide lawful authority  provides for order in schools
o Knoll thinks this is a fiction to satisfy the demands of the COLLINS TEST
Notes:
 This case seems to go against the principles in Hunter, Knoll thinks it’s open to dissent.
Take Away:
1. Charter applies to school searches (148)
2. Vice Principle NOT acting as agent for police (would have been a problem if they were)
3. Students in a school setting have a diminished expectation of privacy
4. School searches are not prima facie unreasonable even though they are warrantless (same as
SIA) 2nd exception to Hunter
5. Collins 1st criteria of search authorized by statute or common law: satisfied by School Act??
6. 4 Rules for school searches: p152
o Warrant is not essential for students under school’s authority
o Need reasonable grounds to believe there has been a breach of school regulations or
discipline and that a search of a student would reveal the breach

R v Tessling 2004 SCC


FACTS:
 FLIR technology can identify heat from above. They were flying in a plane above looking for heat
to identify a grow op.
 Confidential sources said that the accused was running a grow op
 Police took FLIR images from planes and found heat indicative of a grow op
 There was a grow op

ISSUES:
 Whether FLIR images violated the accused’s reasonable expectation of informational privacy. NO

LAW:
 A search is not a search unless it infringes on a reasonable privacy interest.
 3 kinds of privacy:
o (1) Privacy of the person
 Strongest constitutional claim  bodily integrity
o (2) Territorial privacy
 Hierarchy  home is the highest
 Hierarchy is an analytical took to evaluate the reasonableness of a person’s
expectation to privacy
o (3) Informational privacy
 “The claims of individuals, groups, or institutions to determine for themselves
when, how and to what extent information about them is communicated to
others.”
 s.8 only protects a biographical core of personal information  includes
information that reveals lifestyle and personal choices of the individual

APPLICATION:
 Look at the information generated by the FLIR and evaluate its impact on a reasonable privacy
interest
 Information itself is meaningless  not enough for search warrant
 The information is just a photo of outside of house  does not offer insight into private life and
reveals nothing about biographical core of personal information

TAKE AWAYS:
 1) No REP with respect to FLIR technology in 2004
 2) When this technology improves it may be the case that REP may come to exist
 3) This case does not sit well with Kokesch
o Perimeter search where police were sniffing around house. SCC said this violated their
reasonable expectation of privacy.
o Tessling does the same thing, but from a distance.

R v Patrick SCC 2009


FACTS:
 Accused was making ecstasy – police were investigating
 Seized bags of garbage left for collection – inside were supplies to cook ecstasy
 The bags were on the border of his property

ISSUES:
 Whether the garbage left on the curb for municipal pick up was abandoned, meaning the accused
no longer had a reasonable expectation of privacy with regards to the information inside. YES

LAW:
 (1) TEST – Abandoned:
o Whether the claimant has acted in relation to the subject matter of his privacy
claim in such a manner as to lead a reasonable and independent observer to
conclude that his continued assertion of a privacy interest is unreasonable in the
circumstances.
 (2) Consider privacy in terms of the area or thing being searched and the potential impact
of the search on the person being searched, not the nature or identity of the concealed
items.
 (3) Subjective test: whether the applicant had, or is presumed to have had, an expectation
of privacy

APPLICATION:
 (1)AP had interest in the garbage itself and in the informational content of it
 (2) AP had subjective expectation of privacy
 (3) Expectation was objectively reasonable:
o It is not the location of the bags that determines this, but the fact that the police were
trying to find out what was going on inside his home by looking through the bags
o S.8 protects people not places
 (4) The subject matter had been abandoned once it was left for pickup
o He had done everything required of him to give the garbage to the dump
o This would not apply if it was on the porch  it’s when the person has done everything
necessary to have the garbage be collected

TAKEAWAYS:
 (1) Patrick provides for principle of abandonment based on the totality of the circumstances
 (2) Patrick provides for highly qualified example of abandonment RE: property line and PO
reaching over but not coming onto the property
 (3) Garbage is considered to be very important in the context of valuable evidence
Class Notes
 para 21: description of items left in the garbage
o boxes found in a garbage pail in a laundry room
o Demarcation of garbage – totality of circumstances
 Use on exam: “Abandonment is an issue of fact” at para 25

1. Abandonment is a question of fact and is inferred from the conduct of P. He ceased to have a
reasonable expectation of privacy when he put his garbage out for collection and the police didn’t
have to wait for the garbage collector before seizing it. Nor is there any expectation of privacy in the
garbage collector/householder relationship. Abandonment is a function both of location and P’s
intention

ii. Territorial intrusion was peripheral and better seen as pertaining to informational privacy (i.e.
property line not critical)

R v Kang-Brown SCC 2008


– sniffer dogs
(5:4) The police possess a common law power to search using drug sniffer dogs in bus stations
based on a Charter compliant standard of reasonable suspicion.

ii. (Unanimous) The dog sniff of the bag at the bus station amounted to a search within s. 8 of the
charter

iii. (6:3) The dog sniff search of the bag at the bus station violated s. 8 of the Charter.

iv. What will constitute adequate grounds for “reasonable suspicion” is unclear, since the majority is
divided on whether these circumstances were enough.

Per LeBel, Fish, Abella and Charron JJ.: The use of sniffer dogs in these circumstances constitutes a
search under s. 8; it’s not authorized by statute. The common law authority, based on the general
duty of police to investigate crime, requires the standard of “reasonable and probable
grounds”, which was not met in this case. The search, therefore, amounted to a breach of s.
8….the creation of a new and more intrusive power of search and seizure should be left to
Parliament to set up and justify under a proper statutory framework. i.e. these three did not
agree to the “reasonable suspicion standards”

Per McLachlin C.J. and Binnie J.: A “sniff” amounts to a s. 8 search because of the significance and
quality of the information obtained about concealed contents. However, because of the minimal
intrusion, contraband-specific nature, and pinpoint accuracy of a sniff executed by a trained and well-
handled dog, such “sniff” searches should be permitted on a “reasonable suspicion” standard.
The sniff in this case was an unreasonable search since the officer did not have adequate grounds.
“Suspicion” is an expectation that the targeted individual is possibly engaged in some criminal
activity. A “reasonable” suspicion means something more than a mere suspicion and
something less than a belief based upon reasonable and probable grounds. Here, the police
action was based on speculation. Had the dog-sniff search been based on reasonable suspicion, the
dog’s positive alert in this case would have given the police the grounds to proceed on the spot with a
warrantless search of the accused’s bag. The arrest in this case was premature. The police should
first have confirmed the presence of narcotics by a hand search.

Per Deschamps and Rothstein JJ. (dissenting): The use of a sniffer dog in these circumstances
based on “reasonable suspicion” was not an unreasonable search or seizure. The accused had a
reasonable expectation of privacy that engaged s. 8 of the Charter. This is not a case in which the
police were relying on their own senses. Rather, they used the dog to obtain information about the
possible presence of a controlled substance inside the accused’s bag. This involved a certain intrusion
on informational privacy and territorial privacy. Nevertheless, the accused’s objectively
reasonable expectation of privacy was not high. The search was conducted in a public place
and the technique employed was only minimally intrusive. In these circumstances, the
reasonable suspicion standard constitutes a necessary, and therefore justifiable, use of police
powers because it strikes an appropriate balance between the accused’s reasonable privacy
interest and society’s interest in interdicting illicit substances carried on public
transportation. The police had reasonable suspicion in this case and the search was lawful.

Per Bastarache J. (dissenting): The dog sniff constituted a search within the meaning of s. 8. The
accused had a reasonable, but limited, expectation of privacy in his luggage. A subjective expectation
of privacy is evidenced by the protective manner in which the accused carried his bag and his refusal
to allow a voluntary search to occur. Objectively, the odour identified by the dog was not accessible
to humans and its detection provided immediate information about the contents of the accused’s
luggage. A sniffer dog search in this case is not a s. 8 violation where the police are acting on a
“reasonable suspicion”. Reasonable suspicion can be “individualized suspicion”, as in this
case, or it can be a “generalized suspicion”. Police could have used sniffer dogs to search the
luggage of all of the bus passengers, providing they had a reasonable suspicion that drug
activity might be occurring at the terminal, and reasonably informed passengers were aware
that their baggage may be subject to a sniffer-dog search.

CHARGING, ARREST AND DETENTION

1. POWERS OF ANY INDIVIDUAL


 P.231  chapter 3
 Anyone can lay an information
 Information is laid/sworn under oath  process hearing
 s.9 protects against arbitrary imprisonment or detention.
 Any individual can lay an information in writing, sworn by that person who is alleging the
wrongdoing. The Crown can then decide if there is enough information to proceed and press
charges. Process is not automatic, like they are for the police.

(A) INFORMATION BEFORE A JUSTICE: SUMMONS OR


ARREST
R v Pilcher 1981 Man Prov ct Test for laying information
FACTS:
 2 members of the Winnipeg police force were charged
 Wasn’t given the police file to lay the information  just laid the information without the file
 Information was quashed as he did not have sufficient grounds to lay the information
o Was departure from his normal practice  just did this because he was told to it

ISSUES:
 Whether the office had sufficient RPG to arrest based on being told the RPG existed. NO
o He did not read the reports. He only had instructions from a senior official and therefore
he did not have RPG to lay the information
o Reading the report of the investigating officer would have been enough

LAW:
 Officer has to have RPG to arrest someone, person who is laying the information also needs to
have PRG to lay the information  get the background on what is going on
o Doesn’t have to be the person who arrested the person
o A police officer can diligently read the reports of another police officer and write a
lawfully laid information
 S.504 of code now  have to familiarize with the particulars of the case to get RPG 
minimum basis for laying information

APPLICATION:
 Has to have read the report to lay the information  this would give him the RPG to lay the
information

TAKEAWAYS:
 1) Information sworn must be based on RPG
 2) Process must be objective process
 3) RPG are not obtained when someone simply orders a PO to lay an information or arrest
a person
 test for laying information bottom of page 234

R v Jeffrey 0976 ON PC J of P must engage in judicial consideration


FACTS:
 Detective comes in before JP and wants summons issued
 Instead of holding judicial inquiry which is required under code
 Application to quash proceedings  succeeds

TAKEAWAYS:
 1) There must be strict compliance with s.507 of the code for lawful issuance of
summonses
 2) s.507 is not a rubber stamp process
 top of p.236 &p.239 “hears and consider” are the requirements for swearing an
information

Re: Buchbinder and Venner 1985 ONCA


FACTS:
 Police wanted to get Venner  wanted to put him under oath didn’t have enough evidence so
swore an information against an unknown person
 Had justice issue summons against Venner  were going to have a hearing

LAW:
 Process under s.507 is not an investigative process  can’t use the provision to further an
investigation
CLASS
This turns the criminal process into a judicial investigation
1. PO cannot use 507 as an investigative tool
2. PO cannot use 507 to do an end run investigation
(B) ARREST WITHOUT WARRANT
CRIMINAL CODE S.494(1)
494(1)
 Any one may arrest without warrant
(a) a person whom he finds committing an indictable offence; or
(b) a person who, on reasonable grounds, he believes
(i) has committed a criminal offence, and
(ii) is escaping from and freshly pursued by persons who have lawful authority to
arrest that person.

2. SPECIAL POWER OF ARREST OF OWNERS,


POSSESSORS AND THOSE THEY AUTHORIZE
CRIMINAL CODE 494(2)
 S.494(2) extends the power of arrest when it has to do with property offences  if you’re owner
or person in lawful possession
o Not just indictable offence

3. POWERS OF PEACE OFFICERS

(A) BREACH OF THE PEACE


CRIMINAL CODE S.31
 S.31: Breach of the peace
o If PO sees crowd gathering and there is potential for violence, can take action before this
has occurred

(B) ARREST WITHOUT WARRANT


CRIMINAL CODE S.495
 S.495: PO may arrest without warrant person who has committed:
o (A) Indictable offence / has committed or he believes on reasonable [and probable]
grounds, be believes has is about to commit offence or who on
o (B) Person he finds committing [OR APPARENTLY COMMITTING] a criminal offence
 Apparently committing is sufficient
 Any offence under the code
o (C) Reasonable and probable
 What was required in Hunter  constitutional minimum standard for search warrant
o Would not be sufficient – hunch/belief
Class:
 If the arrest falls, the case falls. The arrest is critical

R v Storrey 1990 SCC


LAW:
 RPG is required when getting a warrant
 RPG is very much required to do a warrantless arrest
 Subjective and Objective components  PO must subjectively believe there was RPG, that
RPG must be objectively reasonable.

TAKEAWAYS:
 (1) CC requires arresting officer must subjectively have RPG on which to base arrest
 (2) Must be justifiable from objective point of view
o What were the objective factors for the arrest  apart from hunch
o Reasonable person in place of officer would conclude that there were RPG
o Objective factors should be identifiable
 (3) Doesn’t require proof BRD  doesn’t have to be sure that there will be a conviction  don’t
need to establish a PF case for the arrest
 S.495  have deleted the Probable from RPG
o However SCC has continued on and on to articulate this as RPG  even if not in
section has become part of the CL
 Last paragraph: “In summary the, the CC requires that an arresting officer must
subjectively have reasonable and probable grounds on which to base the arrest. Those
grounds must, in addition, be justifiable from an objective point of view.” (242)

R v Biron 1976 SCC


RATIO:
 The wording in s.495(1)(b) means that the power to arrest without a warrant is given where the
PO himself finds a situation in which a person is apparently committing an offence.
FACTS:
 The police made an authorized raid on a bar in search of firearms and liquor.
 Accused was at the bar while the raid was taking place. He had been drinking.
 He refused to cooperate with the police, verbally abusing them and refusing to give his name. 
o KNOLL: citizens have no legal obligation to do or not do any of these things
 Accused was arrested for causing a disturbance in a public place by shouting, they led him
outside for questioning and he protested this arrest and a scuffle with a Constable occurred. 
o KNOLL: There was no indication that the Accused had shouted (basis for CA acquittal)
 Charged with resisting arrest and causing a disturbance
 The causing a disturbance charge CA acquitted for bothSCC convicted on resisting arrest

ISSUES:
 Whether the resisting arrest charge must fail because the causing a disturbance charge did. NO
 Whether arresting without a warrant requires a subsequent conviction. NO

LAW:
 S.459(1)(a) allows warrantless arrest when officer believes has or will commit indictable
offence  requires reasonable probable grounds, officer has not actually seen the offence
being committed
 S.459(b) requires the officer to have seen the offence being committed  doesn’t require
RPG
o This does not have to lead to a conviction  merely has to be an APPARENT
offence

APPLICATION:
 Although there was not a conviction, the officer was justified in thinking he observed an
apparent offence because the TJ found that he had caused a disturbance  was only acquitted
because the information charged only “causing a disturbance”
TAKEAWAYS:
 (1) Pre-Charter Biron remains alive and is still good law in Canada
 (2)Even if the predicate offence was unlawful, a subsequent related charge may stand
o Analysis makes no sense. “In my opinion the arrest of Biron by Maisonneuve was
lawful.” (p244) If the underlining reason for arrest was unlawful, how was the
subsequent charge lawful?
 (3)Apparently committing is a sufficient nexus for a lawful arrest under s.495
 (4) Refusal to cooperate with the police, refusing to give your name is not an offence
o (Dedman #2, the Dickson dissent: a person does NOT have to respond to police
questioning)
 (6) The Storrey test, reasonable and probable grounds, is subject to an objective criteria
 (5) A person can be considered to be committing an offence when they are APPARENTLY
committing an offence.
o 244: “In the present case Constable Maisonneuve observed an APPARENT offence being
committed…”

(C) CONSTITUTIONAL MINIMUM STANDARDS


CHARTER SS. 9, 10(A)
 Grant made clear unlawful detention is necessarily arbitrary under s.9
 Under s.10(a) everyone has a right “on arrest or detention” to be informed properly of the
reasons therefore.
 Mirrors developments in s.8  for detention to be non-arbitrary has to be authorized by law that
is not arbitrary
o Arbitrary law can be saved by s.1
 Arbitrarily conducted detention cannot be saved by s.1 as it is not proscribed by law

(D) STOP POWERS AND RACIAL PROFIING

(I) VEHICLE STOPS


ARBITRARY:
 Checkstop pullovers have no justification or cause  who the police stop is arbitrary
 Authorized by s.216 of the ON Highway Traffic Act

Hufsky
 Random stops are arbitrary because there is no criteria for selection – violates s.9
 BUT it’s justified by s.1 (Oakes test) because of gravity of issues of motor vehicle accidents,
proportional to the intrusion, and driving is a licensed activity

Ladouceur
 Police were staking out a house – stopped a car that was not related that was passing by to check
license and registration  then arrested for drunk driving
 Was found to be arbitrary, but necessary because can’t set up road blocks in rural areas
 LAW:LIMIT ON WHAT POLICE CAN DO:
o Police can stop vehicles to check sobriety, licensing (ownership, registration) and
mechanical vehicle fitness, but can only ask questions relating to driving offences

Mellenthin
 Police stopped individual – asked what was in his gym bag  there was hash oil
 The visual inspection inside the car was fine, but the questioning about the gym bag was
improper as there was no reasonable suspicion about drugs or alcohol in the vehicle
 LAW: police can only do random stops to check for sobriety, license, ownership, insurance and
mechanical fitness of cars  any other questioning is improper
 Complete Acquittal under 24(2) you can’t ask about a bag in a car for a checkstop

Take Away:
 (1) Vehicle stops may be arbitrary and violation of s.9, but are saved by s.1
 (2) Therefore, vehicle stops may be lawfully engaged in by the police, but there are limits.
 (3) The police can question with respect to the following manners: drivers license, ownership
and insurance, mechanical condition of the vehicle, and to check for sobriety

HIGHWAY TRAFFIC ACT S.216

R v Nolet SCC 2010


FACTS:
 Trucker stopped because his registration was wrong  also did not have fuel sticker
 PO found that his log books were not sufficient, searched the cab  found duffle bag with
$115,000 – arrested for proceeds of crime
 Searched sleeping area  found 392 pounds of marijuana

ISSUES:
 Whether the detention was authorized by statute? YES
 Whether the detention was carried out in a reasonable manner. YES

LAW:
 S.63(5) of the H&TA authorizes PO to search if he has reasonable grounds to believe a vehicle is
being operated in contravention of regulatory requirements.
 Detention has to be authorized by law (statutory or CL) and carried out in a reasonable manner
with respect to the reasonable expectation of privacy.

APPLICATION:
 (1) REP in sleeping area? YES
o But diminished REP because it is not just an area for rest but also a place of work
 (2) Authorized by statute?
o Had reasonable grounds to believe was operating contrary to regulations since
the log books were out of order, the fuel sticker was missing and the license was
wrong
 (3) Carried out in reasonable way?
o The lawful search was not converted into an unlawful one because the officer had
the dual goal of finding drugs  can have two goals in mind and have a legitimate
search so long as the actions don’t violate the charter

TAKEAWAYS:
 1) With spot check search of commercial vehicle the law may allow for greater search than
non-commercial vehicle
 2) A warrantless search of commercial truck or vehicle may be allowed by statutory
provision
o KNOLL: Under each provinces Highway and Safety Acts there are a lot of provisions
surrounding commercial vehicles. These violate s.9 but are saved by s.1 because of the
seriousness
 3) There are minimal privacy expectation rights in the cab of a commercial vehicle  low
REP
 4) A random stop or seizure of a commercial vehicle may not be an arbitrary or un-
savable search re s.8&9

(II) INVESTIGATIVE DETENTION


R v Brown Ont CA 2003
FACTS
 Brown was a young black man in Toronto, was “driving while black”
 PO said they pulled him over for speeding, no evidence
 Accused was charged with driving over .08
 Was acquitted based on arbitrary detention s.9
LAW:
 DEF racial profiling – targeting individual members of a racial group on the basis of the
supposed criminal propensity of that group
 TEST s.9: whether the police officer who stopped the motorist had arti
 culable cause/grounds for the stop  colour is obviously not a valid articulable cause .
Articulable cause exists where the grounds for stopping the motorist are reasonable and
can be clearly expressed…
 Test is balance of probabilities on party alleging the breach

R v Mann 2004 SCC


FACTS:
 Break and enter reported, police found a suspect who seemed to match the description of the
person who committed it a few blocks away from the crime scene 5”8, aboriginal, same
clothing, said this “matched him to a tee”
 Detained and searched – while doing pat down felt something soft in his pocket  was bag with
marijuana
 Court was not satisfied that consent had any part in the search

ISSUES:
 Whether common law authorizes investigative detention. YES
 Whether common law authorizes search incident to investigative detention. YES – but only for
weapons. The item in his pocket did not feel like a weapon, it was a soft object
 Whether at common law the investigative detention was authorized. YES
 Whether searching in Mann’s pocket for the baggie of marijuana overstepped the search incident
investigative detention power. YES

LAW:
 A) Waterfield TEST for common law powers:
o 1) Whether the police conduct falls within the general scope of a duty imposed on
the officer by statute or at common law
 CL duties: preservation of the peace, prevention of crime, protection of life
and property
o 2) Whether such conduct involved an unjustifiable use of powers associated with
that duty
 B) Police officers may detain for investigative purposes if there are reasonable grounds to
suspect that the individual is connected to a crime and that such detention is necessary
o TEST CHANGED: REASONABLE GROUND TO SUSPECT
 Lower than reasonable ground to believe for arrest
 C) Where PO has RG to believe their safety or that of others is at risk, the officer may do a
pat down search of the detained individual
o Has to be connected to safety
o TEST: is it logically possible that someone could have a weapon
o The pat down search must be conducted in a reasonable manner
 D) The detention and pat down have to be done in a reasonable manner
 E) Detention should be brief and subject does not need to answer any questions
 Delay without serious psychological or physical restraint is not detention
Section 10(a):
 At a minimum, individuals who are detained for investigative purposes must therefore be
advised, in clear and simple language, of the reasons for the detention.
 To summarize: KEY POINT
 Police officers may detain an individual for investigative purposes if there are reasonable
grounds to suspect in all the circumstances that the individual is connected to a particular
crime and that such a detention is necessary.
 To summarize:
 In addition, where a police officer has reasonable grounds to believe that his or her safety
or that of others is at risk, the officer may engage in a protective pat-down search of the
detained individual. Both the detention and the pat-down search must be conducted in a
reasonable manner.
 In this connection, I note that the investigative detention should be brief in duration and
does not impose an obligation on the detained individual to answer questions posed by
the police. The investigative detention and protective search power are to be
distinguished from an arrest and the incidental power to search on arrest, which do not
arise in this case.

APPLICATION:
 1) Had reasonable grounds to suspect an offence was committed (Less than RPG), ok to detain
o Proximity to crime scene
o Matching description
 2) Had reasonable grounds to search
o Breaking and entering – could have tools that could be used as a weapon
 3) Searching further to find the drugs was not warranted  admitted the object felt was soft 
not weapon like

DISSENT:
 Use articulable cause instead of reasonable grounds so as to not confused with the standard for
reasonable grounds for arrest
 Search incident investigative detention need not be limited to officer safety  need only be
rationally connected to the reason for the detention
o In this case wasn’t  talking about B&E not drug possession

Judge lefever, allan, fradsham.


(III) ROADBLOCK STOPS
R v Clayton SCC 2007
FACTS:
 911 call that there were 10 black men in a parking lot with handguns
 Police came minutes later, created block to search all cars leaving the parking lot
 Searched two black men in a car that did not match the description  found handguns on them

ISSUES:
 Whether the detention and search fell within the powers of the police stemming from common
law duties. YES
 Whether the detention was based on reasonable grounds. YES
 Whether the detention was reasonably necessary. YES
 Whether the search was to protect officer safety and therefore authorized. YES

LAW:
 Whether detention and search are reasonable will depend on the totality of the circumstances.
 TEST detention: whether there was reasonable grounds to believe the subject was connected to a
crime and whether the detention was justified (reasonably necessary)
 Where is the authority found? Waterfield  Dedman 2 ANCILLARY POWERS: Power from duty

APPLICATION:
 (1) Reasonable grounds – information from the 911 call
o Detention took place within minutes, only those leaving parking area were restricted,
exists of the parking lot were where the people with the guns would go
o Reasonable grounds to believe public safety was at risk  guns in public place
 (2) Whether detention and search were justified
o Even when the cars didn’t match, the subjects were black (matched the description of
the 911 caller)  reasonable grounds that they were involved in the crime
o Officer’s safety justified the search incident to investigative detention

TAKEAWAYS:
 (1) Roadblock stops may be lawful where serious offence has occurred  here was firearms
offence
 (2) Roadblock stop does not have to be narrow and specific, is blanket power  power to stop
every vehicle
 (3) RB stop law arises from ancillary powers doctrine  duty may equal power
 (4) If officer safety is an issue in RB stop that will allow for officer safety search of a person  pat
down for weapons
o Specifically discussed on bottom 2 paragraphs of p.279  reference to Mann
 Searches will only be permitted where the officer believes on reasonable grounds that his
safety, or others, is at risk.
 Look to:
 i. seriousness of offence
 ii. info known to the police
 iii. extent of detention and whether it was responsive or tailored to
the circumstances, including geographical and temporal scope

(E) REASONS FOR ARREST


Section 10 (a)
Establishes the right of everyone “on arrest or detention (a) to be informed promptly of the reasons
therefor.”
CRIMINAL CODE S.29
 S29(2) Duty of everyone who arrests a person, whether with or without warrant, to give notice
to that person, where it is feasible to do so, of the
o (a) process or warrant under which he makes the arrest
o (b) the reason for the arrest

R v Evans 1991 SCC


FACTS:
 Police initially arrested brother for brutal murders  arrested Evans for trafficking marijuana
 Switched to Evans as the suspect – did not explicitly inform him this had changed
 Evans had mental capability of 14 year old

ISSUES:
 Whether it is required to formally inform someone what they are being investigated for. NO
 Whether it is reasonable to infer that Evans knew that he was being investigated for the murders.
YES

LAW:
 It is the substance of what is understood rather than the formalism of how it is said that
informs whether 10(a) is satisfied.
 TEST: whether what the accused was told was sufficient to permit him to make a
reasonable decision to decline to submit to arrest, or alternatively, to undermine his right
to counsel under s.10(b)
 2 purposes of 10(a)
o 1) Have to know what you’re submitting to arrest for
o 2) Inform how you instruct counsel

APPLICATION:
 It became clear he was a suspect – accused responded in interrogation in a way that indicated he
understood this

DISSENT:
 Because he had subnormal intellect needed to be informed explicitly since he was explicitly
informed he was arrested for drug trafficking

R v Greffe 1990 SCC

• The violation of s. 10(a) and (b) resulted in a gross infringement of the accused's rights
pursuant to s. 8 to be secure against an unreasonable search. It is ludicrous to state it, but
state it I must, the rectal search on an arrest for outstanding traffic warrants is totally
unreasonable. The accused accordingly is in the same position as he would have been in if
his rights under ss. 8, 10(a) and (b) had been totally ignored.

(F) ENTRY INTO PREMISES


CRIMINAL CODE S.529.3
 S.529.3 Exigent circumstances exception for warrantless entry:
o (a) RG to suspect entry is necessary to prevent imminent bodily harm or death
o (b) RG to believe evidence will be lost/destroyed imminently without entry
R v Cornell 2010 SCC
FACTS:
 Police did hard entry into premises with masks and battering ram – did not announce their
presence
 Suspected cocaine traffickers  instead found mentally challenged brother of accused
 Possessed cocaine  arrested for possession

ISSUES:
 Whether the police had reasonable grounds for concern to justify the use of unannounced forced
entry while masked. YES

LAW:
To be reasonable under s. 8 of the Charter,
a search must be:
1. authorized by law;
2. the authorizing law must itself be reasonable; and
3. the search must be conducted in a reasonable manner: [Collins 1987 SCC].

 Police normally have to:


o 1) Announce their presence by knocking or ringing doorbell
o 2) Give notice of authority  identify as law enforcement
o 3) Notice of purpose  lawful reason for entry
 Departing from this requires grounds to be concerned about the possibility of violence or
the possibility of the destruction of evidence  onus is on PO/crown to explain why they
thought was necessary to do so
o This should be evaluated from what they knew at the time

APPLICATION:
 Those who traffic in cocaine are often violent  cocaine is easy to destroy
 A lot of latitude for officer safety and destruction of evidence

Strong Dissent

R v Godoy 1998 SCC


FACTS:
 911 call was cut off before the nature of the problem could be found out
 Godoy wouldn’t let police into the house and said everything was fine  police went in anyways
and found his common law partner crying, had been punched in face

ISSUES:
 Whether the discharge of the common law duty to protect life entitles police to forcibly enter a
dwelling. YES

LAW:
 If police conduct constitute a PF interference with liberty the court must consider:
o 1) Does the conduct fall within the general scope of any duty imposed by statute or
recognized at CL
o 2) Does the conduct involve an unjustifiable use of powers associated with the
duty
 Police have power to enter only insofar as it relates to the common law duty to protect life
 anything else would overstep the authority given at CL
 R v Law - Plain view doctrine
o Police had to go through translation process  wasn’t inadvertent finding
o If in plain view also has to immediately obvious/advertent
o Need unaided use of senses to rely on plain view doctrine
o Applies only if obvious and discovered inadvertently

APPLICATION:
 (1) Statutory authority
o The Police Act
o Not necessarily engaged in all 911 calls since not all 911 calls relate to criminal activity
 (2) CL authority
o Duty to protect life  not just related to criminal acts
 911 call is a distress call  whenever there is a distress call the CL duty to
protect life is engaged
 (3) Was it necessary?
o Yes – domestic violence occurs in private/ can’t take the word of the person answering
the door that everything is okay
o Only allowed in so far as connected to protecting life  anything else oversteps

(G) MEANING OF ARREST


R v Latimer 1997 SCC
FACTS:
 Daughter was killed  severely ill and in lots of pain  killed using carbon monoxide
 Police came to his house, didn’t explicitly arrest him, but wouldn’t let him change his clothes by
himself, chartered and cautioned him, and brought him to police station  called it detention
 He confessed at the police station

ISSUES:
 Whether the arrest, though called a detention, constituted a defacto arrest. YES
 Whether s.10(a) was violated because he was not explicitly told that he was under arrest for
murder. YES

LAW:
 Definition arrest:
o (i) the actual seizure or touching of a person with a view to his detention or
o (ii) the pronouncing of the words of arrest to a person who submits to the
arresting officer
 Defacto arrest – it is the substance of what was understood rather than the formalism of
how it is done that determines whether or not there was an arrest.

APPLICATION:
 Defacto arrest occurred through the use of words, conduct of officers and Latimer submitting
 The arrest was lawful – subjectively the officers believed they had grounds despite the fact that
that they didn’t explicitly arrest him / objectively grounds were present
 10(a) – look beyond words used, he clearly knew that it was murder he was being arrested for

Take Aways:
1. SCC in Latimer creates new common law of defacto arrest
2. Even if a person is not notified that they are under arrest and no intention held by
police though even under arrest – they may be under arrest (?? WHY?)
3. If there was no arrest, or no intention to arrest, if it is determined afterwards that
there was reasonable grounds for arrest a defacto arrest may be found
R v Fearon 2014 SCC SIA for Cell Phones
FACTS:
 Robbery, took off with women’s jewelry, fled in black vehicle.
 Later that night, the getaway vehicle was located, 2 men arrested, pat down search found cell
phone. They looked on cell phone, found a text message saying: “We did it were the jewelry at
nigga burrrrrr” and photos, including one of a hand gun
ISSUE:
 If a cell phone is found in an SIA, can the police review the contents of the cell phone? Should the
text message and photos be admitted? YES.
ANALYSIS
 McLaughlin: search did conform to an SIA.
 Common law SIA permits search of cell phones and similar devices, though some limitations are
required.
o 4 part framework:
1. Arrest must be lawful (Stillman)
2. Search must be incident to arrest (Stillman)
a. Protecting the police, the accused, or the public;
b. Preserving evidence; or
c. Discovering evidence, including locating additional suspects, in situations in
which the investigation will be stymied or significantly hampered absent the ability to
promptly search the cell phone incident to arrest;
3. Nature and extent must be tailored to its purpose, only recent photos and text can be
used as evidence
4. Finally, police must take detailed notes
Relevant factors will include:
 the depth of the intrusion. Minor searches may not amount to significant invasions of
privacy, and may be outweighed by law enforcement objectives.
 the importance of the law enforcement objectives. Cell phone searches will more
likely be justified in cases involving violence or public safety. They will likely not be
justified for minor offences.

TAKE AWAY:
 SCC provides for SIA related to cell phones, and provides 4 rules in the context of these searches.
 Protecting police, accused or the public, Preserving evidence, Discovering evidence. – In
situation in which the investigation will be stymied or significantly hampered…
Depth of the intrusion, difference between serious and none serious offences.

INTERROGATION
INTRODUCTION
WHAT IS INTERROGATION?
 Gather evidence in form of statements against accused
 Complex amalgam of:
o Right to counsel s.10(B)
o Common Law Caution (handout)
o Right to silence s.7
 Doesn’t say have right to silence  police doesn’t have to tell you that you have
right to silence
 Our right to silence is articulated in a silent way  not explicit
 Will arise contextually
o Common law confession rule
 Articulated in Card X
 Led to police requirement that they give the common law caution
 ASK: Did the detention crystalize, subjectively and objectively?
o i.e. Cnst Blue psychologically detained the woman when he called at her to come across
the street

WHEN DOES DETENTION CRYSTALIZE? PRE GRANT/SUBARU?


 R v Therens [1988]
o “Detention” in s.10 relates to direct restraint of liberty other than arrest in which a
person may require assistance of counsel
o By demand or direction (Cnst Blue)
o psychological detention
o Necessary element of compulsion or coercion with a demand or direction or a
reasonable belief that one does not have a choice whether or not to comply
 Detention: In addition to deprivation of liberty (1) by physical restraint, there is detention
within 10 (b) when a peace officer (or other agent of the state), assumes control over the
movement of a person (2) by a demand or direction which may have significant legal
consequences and which prevents or impedes access to counsel.
 (3) There is “psychological compulsion in the form of a reasonable perception of
suspension of freedom of choice”.

 R v Terrence
o In addition to deprivation of physical liberty by restraint there is a detention under s.10
of the charter where PO assumes control over movement by demand or direction that
may have significant legal consequences and requires access to counsel
 On top of physical constrain  where people feel they have no choice
o Reasonable belief that one does not have a choice as to whether or not to comply 
matter of choice – has your choice been taken away
o Mann (case for investigative detention) made problem more difficult
 Mann
o Not every detention is a LEGAL detention
o “detained” in the sense of“delayed”or “kept waiting” by police officer is NOT a detention
 however, most people do not know that this is not a detention
o Where is the line between a delay and a detention?
 Grant and Subaru

1. RIGHT TO COUNSEL (CHARTER S.10(B)


S.10(B) CHARTER
S.10 has 2 parts
 Tell why arrested and detained and then give right to counsel
 Retain and instruct counsel without delay

(A) TRIGGERING MECHANISM


R v Grant 2009 SCC
FACTS:
 Three officers on patrol – one in uniform two plain clothes  bad neighborhood (Greenwood
and Danforth) around school, history of student assaults
 Saw accused walking, 18 year old black teen, looking “suspicious” – stared at PO and “fidgeted”
 One officer stepped in his path of him – asked him to keep his hands forward and to explain what
he was doing (uniformed officer)  said had marijuana
 Two other officers moved behind first officer in a “tactical position” – flashed badges
 Asked about previous criminal record, if he had “anything on him he shouldn’t have” and “what is
going on”. Said “keep your hands in front of you” words of control
 He had a firearm (revolver) – was loaded – charged with weapons offences
 Charged on 5 counts (weapons charges)

ISSUES:
 Was Grant Detained within the meaning of ss.9 and 10 of the Charter before the questions that
led him to disclose his firearm? YES
o Detention crystalized right after the officers made it a focused inquiry, and not general.
Specific order = “keep your hands in front of you” made it a detention
o Therefore, 10(b) is triggered, must give a right to counsel immediately
 Violation of 9 and 10(b) rights, not informed of right to counsel immediately,
therefore it was an arbitrary detention
 Should the evidence have been excluded based on charter breach? (covered in evidence law)

Law:
 (1) Detention is determined by choice  per Theren – person is detained when he or she
submits or acquiesces in the deprivation of liberty and reasonably believes that the choice
to do otherwise does not exist
 (2) 2 kinds of detention
o 1) Physical
o 2) Psychological
 A) where the subject is legally required to comply with a direction or
demand, as in the case of a roadside breath sample
 B) where there is no legal obligation to comply with a restrictive or
cohesive
 demand, but a reasonable person in the subject’s position would feel so
obligated
 Focus must be on state conduct in the context of the surrounding
legal and factual situation and how that conduct would be
perceived by a reasonable person in the situation as it develops
 The police conduct/questioning has to attract legal consequences
for the accused
 Length of the encounter is a factor
 Factors:
o Circumstances giving rise to the encounter (eg – general
assistance from police or focused investigation)
o Nature of police conduct (did the accused have no choice
but to comply)
o Particular characteristics of the accused (would a
reasonable person in the circumstances have concluded by
reason of the state conduct…)

APPLICATION:
 (1) Circumstances of the encounter:
o Initially was general inquiries – not detention
o Keep hands in front – not detention – precaution
o Once took tactical position behind, moved to focused suspicion/investigation/
interrogation
 (2) Nature of police conduct  inherently intimidating
o Power imbalance
 (3) Characteristics of the accused
o Very young/inexperienced
 Length of encounter  not fleeting
 Outcome: reasonable person in accused’s position would conclude that his or her right to chose
how to act had been removed by the police given their conduct

The right to counsel arises immediately upon detention, even if it is solely investigative.

Class Notes:
 Para 21: More specifically, an individual confronted by state authority ordinarily has the option
to choose simply to walk away
o Dedman v The Queen – Dickson dissent: Although a police officer may approach a
person on the street and ask him questions, if the person refuses… must allow him
to proceed on his way, unless [he]… arrests him
 Paras 30-32
 TEST: para 44
 Para 46-52 inclusive

TAKEAWAYS:
 1) Facts  facts help understand test
o If analyzing exam question on detention want to frame it in terms of Grant/Subaru
 2) Know para 44  TEST
o Was the focus on a particular specific individual as object of particularized suspicion
 3) Para 46-52
 4) Analysis para 31
 5) Para 52  rights have to be given immediately

R v Suberu 2009 SCC


FACTS:
 Man was trying to use stolen credit card in liquor store – accused was with him
 Accused tried to leave because he was not the one being questioned by the cops – PO said that
he couldn’t leave
o “wait a minute, I need to talk to you before you go anywhere” this was after the PO
followed Subaru outside and asked him that as he was getting into his van
o Man admitted that his friend was the Accused  evidence could be used against him. PO
did not read him his rights at this time.
o PO made no attempt to obstruct his movement
 PO followed him to his van, asked questions
 PO got radio information about the stolen credit cards being used by 2 people in a van 
arrested, chartered, cautioned

ISSUES:
 Whether the accused was detained prior to his arrest. NO
o Taken as a whole, this was a preliminary questioning and NOT an detention

LAW:
 (1) General inquiries do not constitute detention  focused interrogation constitutes
detention
 (2) Factors to determine whether a reasonable person in the circumstances of the accused
would have concluded that he or she had no choice but to comply:
o 1) Circumstances giving rise to the encounter
o 2) Police conduct
o 3) Personal circumstances of the accused
 Suberu established that a s. 10(b) caution must be given “immediately” after a detention
crystallizes.

APPLICATION:
 (1) Circumstances prior to the arrest were of general nature  PO was trying to orient himself to
determine what had happened
o It was only when he received the radio information that he turned towards focused
investigation/suspicion
 (2) Made no move to obstruct movement
o Interpreting the words “I need to talk to you before you go anywhere”  court says that
the PO just wanted more information – purpose of talk
 (3) Did not testify – no indication of personal circumstances

DISSENT (Binnie and Fish)


 Subaru asked to leave, and the POs statement amounted to a “No”
 Discrepancy between Subaru and Grant will lead to uncertainty
o Use these two cases on the EXAM to explain why the facts are either one or the other
 Very narrow gap
o ASK: did the detention crystalize and why? Using para 44 of Subaru

(B) INFORMATIONAL DUTIES


• Purpose of Section 10(b)
• s. 10(b) of the Charter imposes the following duties on state authorities who arrest or detain:
a. to inform the detainee of his or her right to retain and instruct counsel without delay and of
the existence and availability of legal aid and duty counsel;
Implementational :
b. if a detainee has indicated a desire* to exercise this right, to provide the detainee with a
reasonable opportunity to exercise the right** (except in urgent and dangerous
circumstances); and
c. to refrain from eliciting evidence from the detainee until he or she has had that reasonable
opportunity (again, except in cases of urgency or danger).
These duties (b and c)are implementational and are not triggered unless and until the detainee
indicates a desire to exercise their right to counsel

R v Brydges SCC 1990


FACTS:
 Accused was chartered and cautioned, said he couldn’t afford a lawyer and asked about legal aid
 was not given definitive answer and was asked why he would need to speak to a lawyer
 Accused was interrogated, stopped the interrogation to call free lawyer  lawyer said to shut up

ISSUES:
 Whether the accused’s 10(b) rights were infringed by not being told about legal aid. YES
 Whether legal aid availability should be part of the standard charter and caution. YES

LAW:
 If an accused is not informed of the availability of duty council/free lawyer his 10(b) right will be
infringed.

APPLICATION:
 Purpose of 10(b) is to inform accused of their right to silence/right not to incriminate self  this
is evidenced by the fact that he exercised his right to silence after talking to the lawyer

TAKE AWAYS:

1. The obligation on the accused to invoke or assert the right to counsel can be indirect
2. The main function of counsel is to advise the accused to remain silent
3. 10(b) rights include information on legal aid and duty counsel
4. 10(b) caution includes the right to be given details about duty counsel and legal aid

R v Bartle 1994 SCC


FACTS:
 Impaired case – roadside test situation. Failed roadside, was arrested.
 Rights read to accused from caution card – legal aid was mentioned but officer failed to mention
duty counsel/1800 number applicable for jurisdiction
 Accused indicated he wanted to call lawyer, but he didn’t have the 1-800 info  police had the
information but it wasn’t read to him  failed to give him the info
 Distinguish from Brydges: Legal Aid and duty counsel was available at the time of arrest and in
the jurisdiction of the arrest (1-800 number was available)
o Only the law in a rural context, don’t mess this up on the EXAM

ISSUES:
 Whether the information given the accused without the 1800 number was sufficient. NO

LAW:
 (1) Purpose 2.10(b)  provide detainee with legal advice so as to determine how to exercise
rights and fulfill obligations  detainee at disadvantage re: state
 (2) Police have 3 duties under s.10(b)
o 1) Informational Rights – right to counsel (added on by Brydges + Bartel: Legal Aid
and Duty Counsel)
o 2) Implementational Rights reasonable opportunity to contact a lawyer and
stop questioning during the Accused contacting counsel if the A has “asserted the
right”
o 3) Restrain from eliciting evidence
 (3) 10(b) is not absolute accused has to be reasonably diligent about exercising it to
trigger
o “Asserted right”  if the A is silent, there is a context of waiver
 “Unless the detainee invokes the right and is reasonably diligent in exercising it,
the correlative duty ..to provide a reasonable opportunity and to refrain from
eliciting evidence will not arise in the first place or will be suspended” (332)
 i.e. Brydges asked about legal aid, that = an assertion of a right the
correlative police duty
 (4) Understanding:
o Critical that information be given in a clear and comprehensive manner
o Caution must be instructive and as clear and possible
o Detainee does not appear to understand mere recitation of the right is insufficient  PO
have additional obligation to ensure that the accused understands the right they are
being given  if response is questioning the recitation is insufficient
 (5) Waiving right:
o Can’t waive an informational right if you don’t know it exists
o Implicit waiver requires a higher standard than a signed waiver
o Although detainees can waive their s. 10(b) rights, valid waivers of the informational
component of s. 10(b) will be rare.
o

TAKEAWAYS:
 (1) To inform detainee of right to inform/instruct and existence and availability of legal
aid  informational
These rights are not triggered until 1 is engaged
 (2) If detainee has indicated desire to assert the right  if right is asserted  PO have to
provide reasonable opportunity to exercise that right
 (3) Refrain from eliciting evidence until there has been reasonable opportunity
o Once right has been given, PO have to back off
o No assertion – PO can keep asking questions
 (4) PO must give all informational rights in clear and comprehensive matter IE: 1800
number if applicable for jurisdiction
 (5) PO must take additional steps to ensure understanding of those rights if there is some
reason to conclude there isn’t understanding
 (6) Waiver of info rights is rare
 (7) Can’t waive what you don’t know  crown said had waived info about 1800 number
because answered questions

(C) IMPLEMENTATIONAL DUTIES


NOTE CASES:
Pavel:
 Notion that comes out of American TV shows that you get one call
 Must be given reasonable opportunity to make phone call and if mistake is made you have not
had reasonable opportunity to make your call  not mechanical notion of one call
 You get call that reasonably permits you to contact counsel

Playford:
 Accused is entitled to privacy whether they ask for it or not

R v Manninen SCC 1987


FACTS:
 Robbery of Mac’s Milk Store in TO – robber was armed with knife and gun and wore a grey
sweatshirt
 Stolen car
 PO information led to place  saw respondent arrive in the stolen car
 Approached him – identified themselves as PO
 One cop saw gun in the car – took it out
 Arrested, chartered and cautioned  read rights and the accused said he wanted to speak to his
lawyer and would not answer questions
 PO then baited him to answer questions – said only had the gun and not the knife when in the
store, said the grey sweatshirt was his
 There was a phone at the premises that the PO used
 What was read to him at the time?
o Ibrahim caution 1916  Do you with to say anything? Not obliged to say anything
COMMON LAW
 Not obliged to say anything is a form in the common law of right to remain
silent
 Before Charter the common law Caution was the only thing given to the accused

ISSUES:
 Whether continuing to question the accused after he exerted his right to council violated s.10(b).
YES
 Whether not allowing the accused to use the phone on the premises violated his s.10(b) right.
YES

LAW:
 3 components to s.10(b)
o 1) Informational
o 2) Implementation
 Right to retain council has correlative duty on PO to facilitate
o 3) Stop asking questions
 Duty to cease questioning or attempting to elicit evidence

Application:
 (1) Information component satisfied
 (2) Implementation
o No urgency – could have let him use the telephone at the premises
o PO must provide detainee with reasonable opportunity to exercise right and
retain and instruct counsel without delay
 CONTEXT: In this case there was a phone in the shop which the PO could have
allowed the accused to use  violated implementational duty by not allowing
reasonable opportunity to use that telephone
 Accused does not explicitly have to ask for a phone  not asking for the phone
is not a waiver  PO have to offer the phone
 (3) PO continued to question him as though he hadn’t exercised right
o There was not implied waiver  threshold is very high
o Form of questioning was such so as to elicit involuntary answers
o If you continue to question after s.10(b) asserted gives impression to accused that they
have to comply and do not have rights
o Respondent had RIGHT not to be asked questions  not just PO must cease questioning,
accused has RIGHT not to be asked further questions

TAKEAWAYS:
 (1) Right to counsel under s.10(b) includes when the right is asserted a reasonable
opportunity to exercise that right and use of available telephone without need to ask the
PO for use of the phone
 (2) 10(b) right when asserted imposes an implementational duty on the police to cease
questioning  accused has RIGHT not to be asked questions after 10 (b) has been
asserted
 (3) Responding to police questioning after right asserted is not by itself a waiver of 10(b)
rights

Baig v R 1987 SCC


FACTS:
 Accused was arrested and charged with murder – met outside his house
 PO said that he was under arrest and that he had no duty to say anything
 No conversation en route
 Once at station he made a statement incriminating himself
 PO typed it out and had the following questions which the accused answered yes to: do you
understand the charge? Do you understand the caution? I have to inform you that you have the
right to retain counsel without delay.

ISSUES:
 Whether 10(b) was violated by not facilitating right to council. NO

LAW:
 Once the police have complied with s.10(b) by advising the accused without delay of his
right to counsel without delay, there are no correlative duties triggered and cast upon
them until the accused, if he so chooses, has indicated his desire to exercise his right to
council.

APPLICATION:
 He did not assert his right to council, so there are no correlative duties on the police to facilitate

TAKEAWAYS:
 (1) Where PO provide 10(b) rights obligation falls on accused to invoke or assert the right
 (2) If detainee does not invoke or assert the 10(b) right the police may continue to
investigate and question the detainee

Lelclair and Ross v R 1989 SCC


FACTS:
 Youths arrested with B&E – advised of right to counsel and tried to get a hold of their lawyer 
couldn’t
 In one case PO asked if he wanted to get in touch with a different lawyer  Leclair said no
 Then voluntarily participated in a lineup for ID

ISSUES:
 Whether declining to contact a different lawyer waived Leclaire’s 10(b) right. NO
 Whether voluntarily participating in the lineup waived the 10(b) right. NO

LAW:
 (1) Once an accused has asserted their right to council, there need be a clear indication
that he has changed his mind  the burden is on the crown to prove this
 (2) Complying with police demands doesn’t constitute a waiver of right to council 
contrary to the purpose of the section
 (3) Accused persons have a right to choose their counsel and it is only if the lawyer chosen
cannot be available within a reasonable time that the detainee or the accused should be
expected to exercise the right to counsel by calling another lawyer

APPLICATION:
 (1) Informational component is satisfied
 (2) Implementation:
o Accused said he wanted to contact his lawyer, not any lawyer  not wanting to call a
different lawyer doesn’t constitute a waiver
o There was no urgency or other reasons why they couldn’t wait for him to get
satisfactory legal advice
 (3) Refraining from taking further steps
o Lineups – no duty on accused to participate in a line up  reasonable that accused get
legal advice before agreeing to participate
o Lineups – police cannot compel the detainee to make a decision to participate in a
process which could ultimately have an adverse effect in the conduct of an eventual trial
until that person had a reasonable opportunity to exercise the right

TAKEAWAYS:
 1) Reasonable opportunity to contact counsel is circumstances driven
 2) Cease questioning means cease eliciting evidence any form of participation by detainee
 EG lineups
 Right to counsel of choice

R v Brydges 1990 SCC


FACTS:
 Accused was chartered and cautioned, said he couldn’t afford a lawyer and asked about legal aid
 was not given definitive answer
 Accused was interrogated, stopped the interrogation to call free lawyer  lawyer said to shut up

ISSUES:
 Whether the accused’s 10(b) rights were infringed by not being told about legal aid. YES
 Whether legal aid availability should be part of the standard charter and caution. YES
 Whether asking about the availability of legal aid asserted his right to counsel. YES

LAW:
 If an accused is not informed of the availability of duty council/free lawyer his 10(b) right
will be infringed.
 SCC adds on to plain words of 10(b) a requirement to inform of aid/duty counsel

APPLICATION:
 By asking about the availability of legal aid he was asserting his right to counsel  police have
obligation to follow up and make sure this is implemented

TAKEAWAYS:
 (1) Obligation on accused to invoke or assert right to counsel  can be done directly or
indirectly  in effect or in essence
 (2) Main function of counsel when contacted is to advise the accused of the right to remain silent
o Right to remain silent not stated in 10(b) but is part of our law
 (3) s.10(b) rights include right to have information about duty counsel and legal aid
 (4) 10(b) caution must include detail with regard to duty counsel and legal aid

R v Burlingham 1995 SCC


FACTS:
 Accused had already been arrested RE: murder, and was being questioned about the murder of a
second person who was killed around the same time in a similar manner
 Accused continuously said he wouldn’t talk without his lawyer – PO kept saying it would be
better for him and his family to deal with it quickly
 PO also belittled his defence lawyer saying he couldn’t be trusted
 Offered plea – plead to one murder and the other will go away  said the deal was only available
for one night
 Defence lawyer was not available that night – PO said that he could contact a different lawyer
 Confessed
ISSUES:
 Whether the accused’s right to retain and instruct council was violated. YES

LAW:
 (1) Police have duty to advise a suspect of right to council where there is a fundamental
and discrete change in the purpose of an investigation which involves a different and
unrelated offence or a significantly more serious offence than the contemplated at the
time of the original instruction of the right to council.
 (2) 10(b) requires plea bargains be run by defence council absent a formal waiver
 (3) When it is evident that there is a misunderstanding, the police cannot rely on a
mechanical recitation of the right to council – have to take positive steps to facilitate that
understanding

APPLICATION:
 (1) Police continually questioned him after exerted right to council
 (2) 10(b) specifically prohibits belittling of defence council
 (3) Wouldn’t let him talk to HIS lawyer about the plea  allowing accused to call random lawyer
is, given the seriousness of the situation he faced and the changed circumstances in the case,
insufficient for the officers to discharge their responsibilities under 10(b)
 (4) Didn’t let him talk about the plea  no urgency  was trickery  needed to talk to his own
lawyer

TAKEAWAYS:
 (1) PO violated s.10(b) by failing to refrain from eliciting evidence after accused had
asserted right to counsel
 (2) s.10(b) charter is violated when PO belittle defense counsel
 (3) If there is going to be a plea bargain defense counsel must be involved  no
agreements without involvement of defense counsel
 But for the breach, would the evidence have been obtained?

(D) WAIVER AND DUTY TO BE REASONABLY DILIGENT


IN EXERCISE OF RIGHT
R v Clarkson 1986 SCC
FACTS:
 Wife was accused of murdering her husband
 Was very intoxicated when she waived her right to counsel
 Underwent police questioning while very drunk and emotional

ISSUES:
 Whether the accused had sufficient awareness of the consequences of waiving her right to
council. NO

LAW:
 “Aware of the consequences test”:
o not only must an accused person be cognizant of the consequences of waiving the
constitutional right to counsel in a general way, he or she must be aware of the
legal specificities of his or her own case such that there is a presumption against a
valid waiver where the accused is not perceived at the time of the waiver to be
capable of comprehending its full implications.
o Is the accused fully aware of the consequences of giving a statement? If no, it’s a
violation of 10(b) if you do not stop questioning

APPLICATION:
 TJ found as fact that the accused’s confession could not pass the awareness of consequences test
 At minimum PO should have waited until she was sober to properly exercise her right to retain
and instruct counsel or to be fully aware of the consequences of waiver

TAKEAWAYS:
 (1) Test for waiver is on p.354 when person is intoxicated  can person give valid waiver if
intoxicated  not likely  unlikely to understand consequences
 (2) Valid waiver can only occur when the accused can reasonably be said to be aware of the
consequences of their statement
 (3) PO must delay questioning of an intoxicated or impaired person until that person is
sufficiently sober

R v Smith SCC 1989


FACTS:
 Accused was arrested at 7PM RE: a robbery that had taken place 5 months before
 Accused had the police make various stops of a personal nature on the way to the police station
 Exerted his right to counsel – it was 9PM and he determined that he was unlikely to get ahold of
his lawyer at his office
 Said he would wait until the morning, it was 9PM, 2 hours later (differentiate from Lelclair and
Ross, which was at 2AM.)
 Said he didn’t want to answer any questions without his lawyer  police questioned him
anyways
 Made inculpatory statement off the record

ISSUES::
 Whether the accused exercised sufficient due diligence in attempting to contact counsel so as to
discharge the PO’s correlative duty to refrain from questioning. YES
o Accused did not even attempt to make a phone call. Difference between 2AM and 9PM?
Not of note, it was the fact that he didn’t try at all that was fatal.
o Para 256
LAW:
 (1) S.10(b) duties imposed on police (implementation, refrain) are suspended when the
arrested or detained person is not reasonably diligent in the exercise of those rights.
 (2) When an arrested or detained person who has had reasonable opportunity to
communicate with counsel but who was not diligent in the exercise of this right cannot,
subsequently, require the police to suspend, one more time, the investigation or the
questioning.
o This doesn’t apply when the circumstances are substantially different from when
they waived/weren’t diligent.
APPLICATION:
 (1) Decision not to TRY to contact lawyer even though office was likely closed was “fatal”  bars
him from establishing he was reasonably diligent
 (2) Was casual about stopping at different places on the way to the station  this led to him not
being able to contact counsel at a normal time

DISSENT (La Forest):


 Accused positively and repeatedly asserted his desire to exercise right
 Answering questions doesn’t constitute a waiver
 Police actively dissuaded him from contacting
 There was no urgency  had been 5 months since the crime

Take Away:
1) Accused must be reasonably diligent in exercising 10(b) rights in 2 ways:
(a) Must assert the right; and then
(b) Must take reasonable steps to act on it (i.e. use telephone at 9PM)
2) A refusal to even try to contact counsel will lead to further police questioning

R v Sinclair 2010 SCC


FACTS:
 Accused given 10(b) rights and allowed 2 3-minute conversations with his lawyer. He finished
his call.
 Seven hours later interviewed for 5 hours  wanted to talk to lawyer again  wanted lawyer
present. Police said he could choose to remain silent, but could not call his lawyer again and they
would continue the interrogation.
 Eventually confessed
 5-4 split

ISSUES:
 Whether not allowing him to communicate with counsel again, or to have his counsel present for
the interrogation, violated s.10 (b). NO

LAW:
 (1) No right in Canada to have lawyer present during questioning.
 (2) TEST for second legal consultation: where developments in the course of the
investigation make this necessary to serve the purpose underlying 10(b)
o New process (e.g.: lineup for ID)
o Changing jeopardy (victim dies)
o Did not understand legal advice
 (3) Once you have 10 (b) rights police can continue to question you  it’s up to you
whether you want to answer

APPLICATION:
 Test for second legal consultation was not met  there were no changed circumstances
 Two 3 min conversations is enough time to explain the right to silence

DISSENT:
 L’assistance in the French version connoted broader role for legal counsel than simply providing
the advice to keep quiet

TAKEAWAYS:
 (1) Two phone calls of 3 minutes to a lawyer is more than sufficient for 10(b)
 (2) Once the right to counsel has been exercised, the police are free to question for as long
as possible
 (3) 10(b) does not require that a lawyer be present during interview (as in the United
States)
 (4) 10(b) does not provide a right for secondary consultation with lawyer, unless there is
a significant change in the circumstances (p363)
o Level of jeopardy
o New charge
o Change of what is being asked of you (EG: lineup)
2. RIGHT TO SILENCE

(A) VOLUNTARY CONFESSIONS RULE PRIOR TO R V


OICKLE
INTRODUCTION
BASICS COMMONLAW CONFFESIONS RULE
 Accused out of court statement to person in authority needs to be voluntary  CL confessions
rule
 Trustworthiness
o Shown in Wray  verifiable portions allowed
 Privilege against self incrimination
o Where right to silence is overborne  not admissible
 Rothman: Undercover cop in prison  not person in authority
 Ward: Operating mind
 In evaluating voluntariness presence or absence of a caution is only a factor
 Ibrahim v. R  Privy council
o No statement admissible unless shown by prosecution to have been voluntary statement
o Person in authority essential  PO known to be PO

RIGHT TO SILENCE

Nothing in s.10(b) or common law caution say right to remain silent  put in negative context  you
have the right not to say anything

 Common law caution


o Rises out of common law confession rule
 What is our right to remain silent?
o Context is negative  negative
o Right not to have statement given to the police by you used against you unless it is
established that it was given freely and voluntarily (with choice post charter)
o Prior to 1981  wanted to use statement had to have voir dire to determine if the
confession was voluntary
o Post Charter  voir dire but the right has been enlarged, not narrow

Class Notes

 Review from page 365


 Prior to the Charter, the Ibraham rule, from Ibraham (page 370). The Crown had to prove that
an Accused person had made a confession voluntarily
o Normal questions to establish voluntariness: inducements, threats, promises,
violence
 Page 366: An Accused out of court testimony can be used in court if the Crown proves, beyond a
reasonable doubt, that the statement was voluntary. This is where the right to silence comes
from.
 Differentiate from US Miranda right: it’s a negative right, not a positive right.
 Page 370: Ibrahim: No statement by an Accused is admissible unless it is a voluntary statement,
“in the sense that it has not been obtained from him either by fear of prejudice or hope of
advantage exercised or held out by a person in authority.”
o As a result, common law cautions were given out. i.e. Form X Handout.
o Page 340: Everything Under right to counsel is common law: caution to charged
person, secondary caution to charged person
o The common law caution was not necessary, but without it the voluntariness a
statement given by an Accused was highly doubted
 Page 374: If you want to create a new defence, or a variation on an offence, go to section 7 to try
to find a breach. Sections 8-10 are mere reflections of s.7. ss.8-14 address deprivations with
respect to s.7

SALHANY AND CARTER “FUTURE OF THE LAW OF EVIDENCE”

(B) PRINCIPLES OF FUNDAMENTAL JUSTICE (CH. S.7)


Re: s94(2) of the MVA
LAW:
 Principles of fundamental justice is not a right but a qualifier of the right not to be deprived of
life, liberty or security of the person; its function is to set the parameters of that right
 Procedural safeguards + found in the basic tenants and principles of not only our judicial process
but also of the other components of our legal system

R v Rothman 1981 SCC


 Pre-Charter
 Police arrested an Accused, put an undercover agent in the cell with Rothman. A confession
resulted. Court found that it was admissible. Rothman did not perceive his cell mate as a person
in authority, and so Ibrahim did not apply.

R v Henkle 2003 ABCA


Issue:
Whether the breach of the right of disclosure is a separate section 7 Charter? YES
In Stinchcombe, the right to make full answer and defence of which the right to disclosure forms an
integral part was specifically recognized as a principle of fundamental justice included in s. 7 of the
Charter.

R v Hebert 1990 SCC


Rothman revised post Charter
RATIO:
 True choice isn’t just choice to speak to someone you know is a police officer, has to be the
CHOICE not to speak with any PO
 Common law confessions rule stands, and is now enhanced by the Charter. The choice not to
speak to the police that is not respected will be found to breach both the common law confession
rule and s.7 of the Charter.
 The Confessions Rule can be interpreted to be broader than the Charter: “a fundamental
principle of justice may be broader and more general than the particular rules which exemplify it
 378, 379 (until Wray)
FACTS:
 Hebert was arrested for robbery and given s.10(b) rights – declined to make a statement
 Police advised him of his right to silence. He chose not to give a statement to police
 He asserted his right to counsel (who probably told him not to say anything)
 Hebert was put in cell with other inmate who approached him to talk about why he was there
 Make inculpatory statement to the other inmate – was undercover police officer

ISSUES:
 Whether the person to whom the statement was made was a person in authority for the common
law confessions rule. NO
 Whether the statement was admissible under the charter. NO. “The Crown, in using a trick to
negate his decision not to speak, violated his rights.” (387)

LAW
 (1) Right to silence includes s.7/10(b) read together
 (2) Right to silence includes broad right of choice to speak to the police or not – undercover or
not  fairness and integrity in our system of justice is a lawful consideration in that right to
choice
 (3) Rothman is reversed
 (4) Nothing in the rule to prohibit the police from questioning accused after the accused
has spoken with counsel
 (5) Presumably counsel will inform accused of their right to remain silent
 (6) If PO not posing as undercover officers and accused volunteers information will not
violate Charter.
o If accused has got legal advice and counsel
 (7) Right to silence applies only after detention
o Only applies to detention/post detention  undercover operations prior to detention do
not raise the same considerations – are lawful
 (8) Does not affect voluntary statements to other inmates
 (9) If undercover agent is in the cells and is approached by accused who initiates
conversation the admission will be okay  if the police officer initiates the conversation it
will be a problem
o ACTIVELY ELICITING INFORMATION IS THE PROBLEM

APPLICATION:
 Made the choice not to talk to the police
 Officer elicited the information

Scope of Right to Silence (p.385-386)


1. There is nothing in the rule to prevent a police officer from interrogating the Accused after the
right to counsel has bee exercise.
2. Presumably, counsel will advise the Accused of the right to remain silent. If the police are not
posing as undercover police officers, and the Accused choses to provide a statement, it is
admissible.
a. Note: applies only after detention. If a person is NOT under arrest, the common law
confessions rule does not apply.
3. Right to silence does not affect voluntary statements made to cell mates.
4. A distinction must be made by undercover agents observing a suspect, and undercover agents
attempting to illicit information.

Take Away:

1. Common law confession rule, how it arose in Ibrahim and what the rule was.
2. Now the law is a combination of the common law confessions rule and s.7.
3. Rothman is reverse. An undercover police officer in cells may be included in the new
confessions rule. In Rothman they were not.
4. The right to silence includes a right of choice to speak to the police or not. And fairness
and integrity in the system of justice is a lawful consideration.
5. The right to silence includes s.7 and 10(b) read together.

R v Turcotte 2005 SCC


FACTS:
 Accused approached an RCMP station and asked them to go to his farmhouse – would not say
why he wanted them to go there
 When they arrived, 3 people had been murdered with axe
 His silence in refusing to speak about what had happened was used against him at trial.

ISSUES:
 Whether using his silence after the information he gave them was contrary to his rights. YES. At
trial his silence before arrest could be used as evidence against him.

LAW:
 It is wrong to advise jury that silence post act evidence was indicative of state of mind 
RTS requires that silence can’t be used against them
 Would be snare/trap to tell someone they have right to silence and then use that silence
against him

TAKEAWAYS:
 1) RTS is very broad can include circumstances where accused was not in custody or detained
but not absolute right  exceptions: Alibi evidence
 Rule: the right to silence of an Accused means that the exercise of this right can’t be used
in any form against the Accused
 There are exceptions to this rule: bottom of 392.
o Includes alibi evidence. If an alibi is not given in a reasonable and prudent amount
of time, that can be used against an Accused.
 Why? It would be impossible for the Crown to investigate their alibi

Notes:
Chambers case was quoted: It has been well recognized that since there is a right to silence it would
be a snare and a delusion to caution the Accused that he need not say anything in response to a police
officer’s question

(C) NON-COMPELLABILITY (S.11(C)) AND PRIVIELEGE


AGAINST SELF INCRIMINATION (S.13)
Section 13 of the Charter:
A witness who testifies in any proceedings has the right not to have any incriminating evidence
so given used to incriminate that witness in any other proceedings, except in a prosecution for
perjury or for the giving of contradictory evidence.

R v Nedelcu SCC 2012


FACTS:
 Accused took the victim for a ride on his motorcycle – crashed into curb and both victim and
respondent were thrown
 Victim was not wearing helmet and suffered brain damage
 Sued in civil suit – in discovery said that he didn’t remember anything that happened
 14 months later in criminal trial he gave detailed account of the events leading up to and during

ISSUES:
 Whether the statement made during discovery constituted “quid” for which the state owed a
“quo” meaning using it in the accused’s criminal trial would violate s.13. NO

LAW:
 (1) To engage s.13 there needs to be quid pro quo
o Quid: incriminating evidence the witness has given at a prior proceeding in which the
witness could not refuse to answer
 Has to be incriminating
 DEFINITION: evidence given by the witness at the prior proceeding
that the Crown could use at the subsequent proceeding, if it were
permitted to do so, to prove guilt – to prove or assist in proving one or
more of the essential elements of the offence for which the witness
is being tried
 Noel: if the original evidence was not incriminating the quid pro quo was never
engaged
 Time to determine whether the evidence is incriminating is when the crown
seeks to use it NOT when it was first given
 Evidence that could be incriminating if the Crown were to take additional steps
needed to make it so is not enough to trigger the application of s.13  unless
they actually take the steps
o Quo: state side of bargain – state undertaking that it will not use that evidence in any
other proceeding, except in a prosecution for perjury or for the giving of contradictory
evidence.
 (2) The trial judge does not have discretion in whether or not the evidence is incriminating or
not
 (3) Henry did not amend the Charter  still says the evidence has to be incriminating

Thus, a party seeking to invoke s. 13 must first establish that he or she gave [1] "incriminating
evidence" [2] under compulsion at the prior proceeding.
If the party fails to meet these twin requirements, s. 13 is not engaged and that ends the matter.

APPLICATION:
 (1) “I remember nothing” does not go to any elements of the offence and therefore cannot be said
to be incriminating  there is no “quid” which means that s.13 is not triggered
 (2) Use of the non-incriminating evidence for impeachment purposes alone does not trigger s.13
 (3) Only a witness who was lying at an earlier time would be in danger  so long as you’re
telling the truth in your initial testimony it won’t be used against you for impeachment

DISSENT (Lebel):
 Lebel accuses majority of results oriented decision making
 Bargain that is quid pro quo comes from s.5 of the Canada Evidence Act
 Quid pro quo is the historic rationale behind s.13
 Incriminating evidence is any evidence the Crown tenders against the accused
o Even an innocent statement can’t be entered if it helps the Crown
 Compulsion is at the root of quid pro quo
o Compelled means compellable  even though he testified without subpoena he could
have been subpoenaed
 Impeachment vs incrimination is not a workable distinction

TAKEAWAYS:
 (1) If accused gave evidence in previous proceeding that isn’t incriminating it might be
able to be used against them
 (2) S.11  you’re not a compellable witness as an accused person
 s.13 may not apply to civil evidence proceedings if the evidence given is not incriminating
 If the evidence given in a civil procedure (discover, trial, etc) is incriminating, s.13 will
apply

(D) PRINCIPLE AGAINST SELF INCRIMINATION (S.7)

(E) SILENCE AT TRIAL


R v Noble SCC 1997
FACTS:
 Manager of office building sees people breaking into car with screwdriver
 Took the drivers license from the accused, and noted that the person in the photo looked like the
person before him
 At trial the witness could not identify the accused
 The trial judge used the accused’s silence as an adverse inference of his guilty

ISSUES:
 Whether using the accused’s silence as an indication of guilt violated the presumption of
innocence and the accused’s right to silence. YES
 Does the s.13 right to silence forbid a judge from getting evidence? YES. When the Accused
invokes a right to silence, they cannot infer evidence. Alibi can be an exception. If the

LAW:
 The use of silence to help establish guilt beyond a reasonable doubt is contrary to the
rationale behind the right to silence
 The presumption of innocence indicates that it is not incumbent on the accused to present
any evidence at all, rather it is for the Crown to prove him guilty

APPLICATION:
 Trial judge used failure to testify as evidence going to identification for proof BRD  improper

R v Prokofiew 2012 SCC


FACTS:
 2 people involved in a fraudulent scheme to do with sales tax
 the accused did not dispute the fraudulent scheme or their involvement
 question for the jury: were either or both of the accused aware of the fraudulent nature of the
scheme
 2 accused  one gave evidence, other did not
o Counsel for the one who did give evidence said that the co-accused didn’t give evidence
and this is indicative of guilt
o TJ relied on s.4(6) of the Evidence Act
 Both of the Accused were convicted

ISSUES:
 Whether it is improper for one counsel to point to a co-accused’s silence as indicative of
guilt. YES

LAW:
 (1) S.4(6) of the Evidence Act
o Judge can’t comment on lack of testimony  counsel for prosecution can’t comment on
lack of testimony  Canada evidence Act doesn’t say that counsel for accused couldn’t
comment on this
 (2) Defense counsel not entitled to comment on the silence of a co-accused not
withstanding that s.4(6) of evidence act doesn’t cover that
 (3) SCC agreed the judge could right the wrong of what could be done  only can’t comment
prejudicially on this
 (4) evidence act doesn’t prevent a positive comment from the TJ  TJ can only make prejudicial
comment
Take Away:

Section 4(6) of the CEA does not prohibit a trial judge from commenting on an Accused’s right to
silence

(F) RE-STATED CONFESSIONS RULE R V OICKLE


R v Oickle SCC 2000
FACTS:
 Accused arrested in relation to a series of car fires including his fiancé’s car and her father’s
 Submitted voluntarily to a polygraph test, which he failed
 After polygraph test was questioned for several hours and eventually confessed – first to just the
fire in his fiancé’s car and after to all of them
 Police informed him of right to silence
 PO also said that it would be better if he confessed, conflated the accuracy of the polygraph test
 PO lied and said his fiancé was in the next room, indicated it would be better for his fiancé if he
confessed
 Police were pleasant to him throughout, provided the necessities, told him he could leave
 Didn’t tell the police that he wanted to go home until after the polygraph
 Trial judge ruled the confessions were voluntary  court of appeal overturned

ISSUES:
 Whether the police improperly induced the respondent’s confession through treats or promises,
an atmosphere of oppression, or any other tactics that could raise a reasonable doubt as to the
voluntariness of the confession. NO

LAW:
 (1) 4 kinds of false confessions:
o 1) Stress complaint  where adverse interpersonal pressures of interrogation
become so intolerable that suspects comply in order to terminate questioning.
o 2) Coerced-complaint confessions  classically coercive influence techniques
(threats and promises)
o 3) Non-coerced-persuaded confessions  police tactics cause innocent person to
become confused, doubt his memory and be temporarily persuaded of their guilty
 often involves use of fabricated evidence
o 4) Coerced-persuaded confessions  like non-coerced persuaded but also
includes classic coercion tactics
 (2) Themes:
o Need to be sensitive to particulars of accused
o Danger of using non-existent/fabricated evidence
o Emphasis on threats and promises
 (3) Voluntariness can be affected by:
o 1) Intolerable conditions
o 2) Inducements
o **Both can work together  even slight inducements can render a confession
involuntary if the conditions are intolerable
 Need contextual analysis to look at how this all works out
 (4) Standard of review: Voluntariness is a finding of fact if done properly  deference

APPLICATION:
 Was voluntary  was always told of his rights, never subjected to bad conditions, not deprived
of sleep, not offered improper inducements
 (1) Inducements:
o Offer of psych help  not contingent on the confession  no quid pro quo to make it
inappropriate (top of 422)
o Ibrahim rule is expanded from a “hope of advantage” to include “a fear of prejudice”
 It would be better if you confess  moral inducement  no problem
o Threats against fiancé  she was in no danger, not real inducement
 (2) Abuse of trust because they were nice to him? Absurd outcome of making police be harsh in
their interrogations
 (3) Atmosphere of oppression  Rothman  Lamer’s judgment  none of the conditions
brought administration of justice into disrepute
o Since the atmosphere was not oppressive, the minimal inducements that there were are
okay
 (4) Polygraph  they overstated how accurate it was  but confronting a subject with adverse
evidence, even if exaggerated, will not standing alone render a confession involuntary

TAKEAWAYS:
 (1) CL confessions rule from Ibrahim is maintained by the SCC in Oickle, however it has been
fleshed out and updated to include context including threats, promises, oppression, an operating
mind and police trickery
 (2) CL confessions rule applies pre-arrest or detention
 (3) 4 potential areas impacting voluntariness
o Oppression
o Operating mind – appreciates the risk of confession and is aware of the consequences
o Police trickery – it’s ok to lie and make things up, to set up booths giving out gum to
obtain a DNA sample. However, there are limits, such as disguising a PO as a priest and
obtaining a confession, and giving a truth syrum instead of insulin for diabetes
o Lamer (p.426): not a game to be governed by the Marquess of Queen
 (4) Test from 447 is the modern test for confessions
Circumstances overbears the accused’ will.

 Must prove voluntariness BARD

not every inducement will render a confession involuntary.

Spencer 2007 SCC

He offered to confess in exchange for lenient treatment for her. The interviewing officer told the
accused he couldn’t make any deal with him. The accused also requested a visit with his girlfriend
but was told it would only be allowed after he cleaned his slate. The accused confessed to some of
the robberies and was allowed to visit her. He then confessed to the other robberies.
At trial: accused’s statements admitted and convicted of 18 robberies.
SCC: statements admitted
R v Singh SCC 2007
FACTS:
 Arrested in connection with a stray bullet shot in a night club that killed someone
o Photo of him wearing backwards hat from bar security on the night of the shooting
o Photo of him at the same bar a few days later from bar security
 No physical evidence
 Arrested – said 18 times that he didn’t want to talk and wanted to be brought back to his cell
 First of police interview he admits to having been in the bar on the night of the shooting but left
before the shooting occurred
o Then, he admitted to being the person in the photo
 Sargent said he was “determined to get a confession”. Interrogated for 48 hours. No evidence of
oppression.
 TJ deemed that the admissions were voluntary and admitted the evidence

ISSUES:
 Given that the voluntariness of the statements was conceded, did the manner in which the
statements were obtained violate the accused’s s.7 right to silence? NO
o Confessions rule and s.7 are the same in this context. Voluntariness in Oickle applies

LAW:
 (1) Considerable overlap between voluntariness and s.7 right to silence
o Confessions rule and right to silence are manifestations of the principle against self-
incrimination
o Herbert – s.7 right to silence is informed by the confession rule  right not to speak to
the authorities
 (2) FUNCTIONAL EQUIVALENCE:
o TEST for determining common law confessions rule voluntariness and s.7 right to
silence are functionally equivalent  no need for double barreled test of
admissibility
o BUT NOT WHOLY SUBSUMED:
 Common law confessions rule is not wholly subsumed  can still have
situations where it provides greater protection than s.7  EG: Oickle where
police was undercover  the confession was voluntary because he didn’t know
it was a cop, but violated s.7
 Residual
 (3) CL = BETTER PROTECTION:
o Common law affords better protection that s.7 right to silence as the Crown has to
prove voluntariness beyond a reasonable doubt, whereas accused has to prove s.7
violation on a balance of probabilities
o Common law right to silence: reflects general principle that absent statutory or legal
compulsion no one is obliged to provide information to the police or respond to
questioning
o Right to silence has always been part of the common law confessions rule
therefore:
 If BRD the Crown has proven voluntariness s.7 right to silence will not be
infringed
 Finding of voluntariness requires that the court consider right to
silence  right to silence is defined in accordance with constitutional
principles  finding voluntariness decides s.7
 If on BOP accused proves s.7 right to silence is infringed, Crown will not be able
to make out voluntariness BRD
 (4) WARNING ABOUT RTS:
o Presence or absence of a warning about the right to remain silent is just one factor in
determining whether the right has been violated
 (5) PRE/POST DETENTION:
o Standard is different before and after detention as subjects are more vulnerable after
detention

APPLICATION:
 (1) Balancing need to protect accused’s rights and society’s need to investigate crime
 (2) Given that voluntariness is conceded, the s.7 argument must fail
 (3) It is problematic that the PO said he would put parts of the police case to accused to get him
to confess no matter what
o Even if problematic it is a question of degree  this was not egregious enough
 (4) If voluntariness assessed properly it is a finding of fact  reviewable with deference  TJ did
not make a palpable and overriding error

DISSENT (Fish):
 Accused was powerless to end the interrogation  was in detention  totally under control of
police authorities  kept asking to be returned to cell an wasn’t
o When he kept asking and his request kept being denied he was given the impression that
resisting was futile  undermined right to remain silent
 Had to forsake counsel advice (told to keep quiet) violates s.10(b) as well
o S.7 right to silence and s.10(b) right to counsel are thus linked
 # of times he said he didn’t want to talk was not necessary  right has to be respected even if
only said once
 Common law confessions rule is not the same as the s.7 right to silence
o Purpose of CL rule: reliability of confessions + integrity of CJ system
o Purpose of s.7: constrain gov’t action in conformity with rights and freedoms, the
preservation of which are essential to the continuation of a democratic, functioning
society in which the basic dignity of all is respected
 Herbert – s.7 right to silence must be interpreted in a manner which secures to
the detained person the right to make a free and meaningful choice as to
whether to speak to the authorities or remain silent
o Just because a confession is voluntary (per the test from Oickle) does not mean that it
was given in accordance with a free and meaningful choice for the purpose of the charter
APPLICATION:
 Detainee is in custody of state and subject to total control
 When left alone to face interrogators who persistently ignore their assertions to right to silence
are bound to feel their constitutional right to silence has no practical effect and they have no
choice but to answer
 CL allows police to ask questions but constitution protects accused’s rights not to answer them

TAKEAWAYS:
 (1) Repetitive and persistent questioning by the police despite multiple claims of right to
silence does not vitiate a voluntary confession
 (2) once the Accused has exercised their s.10(b) rights the police can interrogate
endlessly without any violation of Oickle or the right to silence. Endless interrogation is
NOT in violation of Oickle.

NEW COMMON LAW IN CONNECTION WITH MR. BIG


OPERATIONS
 Mr. Big operations operate in a legal loop hole:
o person receiving confession cannot be a person in authority for the common law
protection on confessions to be triggered. If it is voluntary and not given to a person
that the Accused knows is in authority, it is a legally, voluntarily obtained
confession. Jail cell confessions protected by Ebert is not applicable because in a Mr.
Big operation the person is not in psychological detention and can obtain legal
advice.
 Only way to exclude: if police conduct would “Shock the community”. However, outrageous
conduct (i.e. Hathaway - they beat up women and put her in trunk) is often not found to be
conduct that would “shock the community”.
 Questions being raised on the reliability of evidence obtained with Mr. Big techniques. Have a
70% confession rate and 90% conviction rate. However, it is very prejudicial for a judge/jury to
hear evidence from a person who (is intoxicated and) confesses (under duress). Transcripts often
show a lot of denials, then after a lot of alcohol, a confession results.
 Police officers get to know the Accused, figure out what they want, and give incentives for
confessions down the road.

R v Hart 2014 SCC 52
FACTS:
 Accused’s daughters drown in a lake – his initial story was that the first one fell in the lake, he
freaked out and went to get his wife, at which point the second one fell in the lake
 Later changed his story to say that he was having a seizure, but didn’t want to disclose because
he didn’t want to lose his license
 Police set up Mr. Big operation – elaborate – he began to see the officers as his family
 Grade 5 education, poor, no connections
 He initially said the seizure story, but when Mr. Big pushed him he said he murdered them
because his brother was going to take the children away  there was no evidence of losing the
children
 Participating in crimes taken by some as corroboration
o Agreed to go to Walmart store while “iceman” was going to “fix” someone who they had
told him had been an eyewitness to the murder
 Inference that he did not say “there’s no reason because I didn’t kill my
daughters” that he killed his daughters  inference from fictional state of mind
 63 scenarios done  after 20 scenarios the police should have been worried  by 29 they knew
that they were in danger  did many more after this  majority said went too far
 VD – Judge ruled admissible  jury convicted  appealed

ISSUES:
 Whether Hart was controlled by the state despite not being detained, meaning his statement
violates his right to silence. YES

LAW (Moldaver; McLaughlin, LaBel, Abella, and Wagner concurring):


 (1) 3 MAJOR CONCERNSwith Mr. Big confessions:
o Danger of unreliable confessions
o Prejudice effect of Mr. Big confessions
o Police misconduct
 (2) OBJECTIVE: 2 Pronged Approach addresses the 2 objectives:
o toprotect an accused's right to a fair trial under the Charter, and
o to preserve the integrity of the justice system.
 (3) 2 PRONGED APPROACH(para 85-86)
1. Where the state recruits an accused into a fictitious criminal organization of its own making
and seeks to elicit a confession from him, any confession made by the accused to the state
during the operation should be treated as presumptively inadmissible. The Crown must
establish, on a balance of probabilities, that the probative value of the confession outweighs
its prejudicial effect.
a. The assessment of probative value involves an assessment of reliability
b. Prejudicial Assessment: considers moral prejudice and reasoning prejudice
2. The doctrine of abuse of process doctrine will be used to deal with police misconduct
a. Onus is on the Accused here  
b. “there are "inherent limits" on the power of the state to "manipulate people and events
for the purpose of ... obtaining convictions" (R. v. Mack)” (para 112)
c. the doctrine provides trial judges with a wide discretion to issue a remedy —
including the exclusion of evidence or a stay of proceedings — where doing so is
necessary to preserve the integrity of the justice system or the fairness of the trial 
d. A confession derived from physical violence or threats of violence against an accused
will not be admissible — no matter how reliable — because this, quite simply, is
something the community will not tolerate (see, e.g., R. v. Singh)
e. Operations that prey on an accused's vulnerabilities — like mental health problems,
substance addictions, or youthfulness — are also highly problematic (see Mack)
 (4) Reliability: Consider the circumstances of the confession and the confession itself
o Look at circumstance of confession:
 length of operation, number of interactions between PO and accused, nature of
the relationship between undercover officers and accused, nature and extent of
inducements offered, presence of any threats, conduct of the interrogation itself,
and personality of the accused, including his or her age, sophistication, and
mental health.
o Look to the confession itself for confirming facts:
 Level of detail contained in the confession, whether it leads to the discovery of
additional evidence, whether it identifies any elements of the crime that had not
been made public (e.g., the murder weapon), or whether it accurately describes
mundane details of the crime the accused would not likely have known had he
not committed it (e.g., the presence or absence of particular objects at the crime
scene)
 (5) Prejudicial Effect:
o Moral Prejudice: jury learns that the accused wanted to join a criminal organization and
committed a host of "simulated crimes" that he believed were real. In the end, the
accused is forced to argue to the jury that he lied to Mr. Big when he boasted about
committing a very serious crime because his desire to join the gang was so strong. Moral
prejudice may increase with operations that involve the accused in simulated crimes of
violence, or that demonstrate the accused has a past history of violence.
 (6) “However, it must be remembered that trial judges always retain a discretion to exclude evidence
where its admission would compromise trial fairness” (para 88)
 (7) APPLICATION: In practice, this two-pronged approach will necessitate that a voir dire be held
to determine the admissibility of Mr. Big confessions.
o The Crown will bear the burden of establishing that, on balance, the probative value of
the confession outweighs its prejudicial effect.
o The defence will establish an abuse of process.
o Trial judges may prefer to begin their analysis by assessing whether there has been an
abuse of process. A finding of abuse makes weighing the probative value and prejudicial
effect of the evidence unnecessary.

APPLICATION:
 Danger of Unreliable Confessions
o Had to confess to stay in culture – had nothing to lose by confessing – had everything to
lose by not confessing
o His credibility is trashed and he is required to testify
o Without Mr. Big evidence there isn’t even a crime
o Mr. Big where admitted has been fact specific
o Community, lifestyle, permanence  combination of factors that made it impossible for
him not to confess
o Pressure – would have confessed even if innocent  would lose everything
o Framing of the question
o Inconsistencies in his confession  internal and external
 Prejudicial effect of Mr. Big Confessions
o Mr. Big confessions are invariably accompanied by prejudicial facts regarding the
accused's character.
o Character evidence is unreliable
 Moral prejudice – risk that the jury will come to conclusions based on the
Accused’s perceived disposition
 Reasoning prejudice – distracts jury’s focus from offence to extraneous acts of
misconduct
 Police Misconduct
o Put lives at risk by allowing Hart to drive a delivery van when he is epileptic
o 63 scenarios done  after 20 scenarios the police should have been worried  by 29
they knew that they were in danger  did many more after this  majority said went
too far

CLASS NOTES:

 (1) Code thematically set up the course to talk about the impact of court decisions re:
admissibility on PO investigation tactics
 (2) “whether by accident or design Mr. Big structured to avoid barriers”
o Set up in a away to avoid the rules of evidence excluding the admission
o ROTHMAN: Right to silence only applied post detention
o What barriers:
 1) Subject doesn’t know person of authority  voluntariness
 Rothman Hebert Grandinetti
 2) Subject not in custody
 Hebert Rothman
 3) Admissions by parties not subject to principled approach
 Crown doesn’t have to show threshold reliability
 4) Abuse of process exclusionary rule
 Test is investigative techniques have to shock the conscience of the
community
 Because Mr. Big has been admitted so many times  impossible that
Mr. Big confessions would be deemed to shock
 Put him to work as a delivery driver because it was his greatest dream
 putting everyone on the road at risk because of his epilepsy 
shocks the conscience of community
 Grandinetti concludes that MR. Big admissions admissibility should be
filtered through the analysis of abuse of process
 (3) Adversity of interests creates stamp of reliability
o Admissions against interest
o But in Mr. Big it is not admitted as an admission against interest  admitted as a party
admission
o Not actually against interest if facts were the way the accused believed them to be
 (4) Statements are hearsay:
o Have reliability problems RE: Baldree
 Mr. Big admissions not subject to Baldree / principled approach
 Party admissions is hearsay but practically because it is an out of court
statement made by a party (the party is in court) – if wants to say that their
prior statement was unreliable can recant
 Ferris  essence of McClung’s dissent
o Open to the party who is in court to say so  being so
compelled isn’t so compelled contrary to meeting case: just the
way the adversary system works
 (5) Tension between providing fair trial to accused and police investigations:
o At most basic appeal is consideration between providing fair trial to accused and not
impeding police investigation
 Any limitation on admissibility of Mr. Big statements might hamper
investigation of crime

Grant SCC 2009


FACTS:
 Accused’s s.8/10(b) rights were violated and a gun was found as a result

ISSUES:
 (1) Clarification of the criteria relevant to determining when, “in all the circumstances”
admission of evidence obtained by a Charter breach “would bring the administration of justice
into disrepute”
 (2) Whether the gun produced should be excluded from the evidence at his trial. NO

LAW:

Section 24(2)
Two preconditions for exclusion:

• 1. The evidence must be "obtained in a manner that infringed or denied any


rights or freedoms guaranteed" by the Charter; and
• 2. The admission of the evidence in the proceedings "would bring the
administration of justice into disrepute”.

 (1) 3 facts to consider, which have to be balanced:


o 1) The seriousness of the Charter-infringing state conduct
 Admission may send the message that the justice system condones serious state
misconduct
 Purpose is not to punish police or deter Charter breaches but to preserve the
public’s confidence in the rule of law
 Extenuating circumstances – need to prevent disappearance of evidence
 Good faith on the part of police VS. willful or flagrant disregard
o 2) Impact of the breach on the Charter-protected interests of the accused
 Admission may send the message that individual rights count for little
 Look at the interests engaged by the infringed right and examine the degree to
which the violation impacted those interests
o 3) Society’s interest in the adjudication of the case on its merits
 Whether the truth seeking function of the criminal trial process would be better
served by admission of the evidence or by its exclusion  the impact of failing
to admit the evidence
 Reliability of the evidence
 If breach (eg. Compelling suspect to talk) undermines evidence  leads
to exclusion
 Importance of the evidence to the Crown’s case
 Seriousness of the offence at issue  limited relevance
o Main concerns:
 Factors 1/2:
 Heightened concern with proper police conduct in obtaining
statements from suspects and the centrality of the protected interest
affected will in most cases favour the exclusion of statements taken in
breach of charter
 Factor 3:
 Reliability frequent concern
 (2) Statements by accused  CL also protects
 (3) Types of evidence:
o Bodily evidence
 Reverses Stillman  no longer exclude all bodily evidence if conscripted
 Reliability will usually favour admission under the third branch
o Non-physical bodily evidence
 Key consideration is extent to which the conduct can be characterized as
deliberate or egregious
 REP
 Human dignity
o Derivative evidence
 Physical evidence discovered as a result of an unlawfully obtained statement
 Discoverability  relevant in the second branch  helps assess the actual
impact of the breach on the protected interest of the accused

APPLICATION:
 Gun was derivative evidence
 (1) Seriousness of conduct:
o Conduct was not in conformity with the Charter but not abusive
 No racial profiling
 The mistake (that he was detained) was an understandable one
 No bad faith
 Not deliberate or egregious
 (2) Consider rights separately:
o A) Arbitrary detention
 No physical coercion but subtly coercive – deprived freedom RE: making
informed choice
 Not severe but more than minimal
o B) Right to counsel
 Incriminating statements were conscripted
 Was in need of legal advice but not told he could consult counsel
o Discoverability – would not have searched but for the incriminating statements  Gun
o Significant breaches
 (3) Adjudicating case on merits
o Gun is highly reliable
o Seriousness of offence is not relevant, in favour of state and accused
o Police conduct was not egregious
o The evidence is very reliable  high value
o Should be admitted.

CLASS NOTES:
 (1) Adjudication of the case on its merits
o Society’s interests in
o MERITS  one of the most important merits is the fairness of the adjudicative process
and the trial fairness
 The first two factors are what protects this
o CODE – to name the third factor this way says the first two factors aren’t“the merits”
 (2) Court’s role is balance all three inquiries to determine if it brings into disrepute  if it does

 it shall be excluded  language of a rule once you get through the IF
R. v. Harrison, 2009 SCC 34
FACTS:
 The accused and his passenger Sean Friesen were driving on an Ontario highway when they
passed Cst. Bertoncello travelling the opposite direction. Noticing that the vehicle was missing a
front licence plate, which for a car registered in Ontario would constitute an offence, the officer
turned on his vehicle lights and pursued the vehicle to pull it over.
 As he approached the vehicle from behind, Bertoncello discovered that the car had an Alberta
licence plate and was therefore not required to have a front licence plate. No other reason to pull
the car over, was driving the speed limit.
 Despite knowing no grounds existed to form a reasonable belief that an offence was being
committed, he pulled the vehicle over. After learning that the accused was driving with a
suspended licence, the officer arrested the accused and proceeded to search the car “incident to
arrest” ostensibly for the missing license.
 Beginning the search with the trunk of the vehicle, Bertoncello discovered two cardboard boxes.
Upon learning that one of the boxes contained a substance resembling cocaine, the officer
proceeded to search the rest of the vehicle, recovering 35 kilograms of cocaine in total.
 The violation of the accused’s s.8 and s.9 Charter rights is not in dispute.

ISSUE:
 With these Charter violations established, the legal issue before the Court is whether the
cocaine should be excluded from evidence under s.24(2) of the Charter.*
o YES

LAW
 The three factors to be balanced are as follows:
o 1)the seriousness of the Charter-infringing state conduct;
o 2)the impact on the Charter-protected interests of the accused; and,
o 3)society’s interest in an adjudication on the merits of the case.
 Public interest must be weighed at every stage of the Grant analysis.

ARGUMENTS:
 Defence:
o Seriousness of Conduct: The officer’s conduct cannot be characterized as unsound
decision-making or a mistaken but honest belief in the legitimacy or validity of his
actions. Every moment during this arrest signifies an abuse of police power, a
blatant disregard for Charter rights, and emphasizes the fact that this officer was on
a mission to find something in the vehicle regardless of the wanton disregard for the
law necessary to complete this very search.
o Impact on the violation: No reasonable expectation of being pulled over, no reason
to. We cannot justify a police officer’s conduct, which negatively impacts a motorist’s
rightful expectation of liberty and privacy purely in the unlikely hopes of securing
evidence which could be used to facilitate a criminal proceeding.
o Public Interest: The conduct of the officer was deliberate and serious, and the
impact on the breach on the Charter-protected interests of the accused was severe.
 Crown: Section 24(2) of the Charter operates to admit evidence obtained through Charter
violations of the police in circumstances where, after weighing all the factors, the need to
dissociate the justice system from police misconduct clearly presents itself.The evidence on
each line of inquiry must be weighed to determine whether, having regard to all the
circumstances, admission of the evidence would bring the administration of justice into
disrepute.
o Seriousness of Conduct: The officer’s conduct was not shown to be systemic in
nature, or the result of operational policies or guidelines, or even an order from a
senior officer. The actions involved were those of one officer, no bad faith, no
malice, not premeditated.
o Impact of the violation on the accused’s Charter-protected interests: The Charter
breaches did not have a particularly serious effect on the accused’s Charter rights. Lower
expectation of privacy in a car, on public property. Privilege of driving brings with it a
duty to other drivers.
o Public Interest:The drugs seized constitute highly reliable evidence tendered on a
very serious charge. This factor weighs heavily in favour admission. In those
circumstances, as the case should be here, the public interest in truth-seeking
rightly became determinative. No malice or pre-mediation by the PO.
APPLICATION
 1) the seriousness of the Charter-infringing state conduct;
o Conduct of the police that led to the breaches represented a blatant disregard for Charter
rights, further aggravated by the officer’s misleading testimony at trial. 
 2) the impact on the Charter-protected interests of the accused;
o Deprivation of liberty and privacy by the unconstitutional detention and search was a
significant, although not egregious, intrusion on the accused’s Charter rights
 3) society’s interest in an adjudication on the merits of the case.
o drugs seized constituted highly reliable evidence tendered on a very serious charge.
However, the seriousness of the offence and the reliability of the evidence, while
important, do not in this case outweigh the factors pointing to exclusion.
o To appear to condone willful and flagrant Charter breaches amounting to a significant
incursion on the accused’s rights does not enhance, but rather undermines, the long-term
repute of the administration of justice. 
 No factor outweighed any other in the Grant assessment. This differed from the ONCA majority’s
decision in that it gave more weight to the impact on the accused’s Charter-protected interests.
 The Charter-protected rights of the accused were impacted significantly.
 Dissent: Deschamps downplayed the Charter breach, proposed a 2 prong approach to Grant that
focuses much more on the public interest

Class Notes:

 What is deliberate action? Which deliberate is bad? Which is good?


 Harrison is important because it was released on the same day as Grant, opposite decision.
Harrison is an important lesson for trial judge: knowledge that the Accused committed the
crime only means that he did it, not that he is guilty. Guilt involves proof

You might also like